Top Banner
INSIGHTSIAS SIMPLYFYING IAS EXAM PREPARATION INSTA Tests 37 to 40 (GS) www.insightsonindia.com prelims.insightsonindia.com | mains.insightsonindia.com Telegram: insightsIAStips | FB: insightsonindia | TW: vinaygb | YT: INSIGHTS IAS BENGALURU | DELHI | HYDERABAD INSTA 75 Days REVISION PLAN UPSC Prelims 2020 Copyright © by Insights IAS All rights are reserved. No part of this document may be reproduced, stored in a retrieval system or transmitted in any form or by any means, electronic, mechanical, photocopying, recording or otherwise, without prior permission of Insights IAS. KEY & EXPLANATIONS
99

SIMPLYFYING IAS EXAM PREPARATION - INSIGHTSIAS€¦ · This document is the compilation of 100 questions that are part of InsightsIAS famous INSTA REVISION initiative for UPSC civil

Jun 14, 2020

Download

Documents

dariahiddleston
Welcome message from author
This document is posted to help you gain knowledge. Please leave a comment to let me know what you think about it! Share it to your friends and learn new things together.
Transcript
Page 1: SIMPLYFYING IAS EXAM PREPARATION - INSIGHTSIAS€¦ · This document is the compilation of 100 questions that are part of InsightsIAS famous INSTA REVISION initiative for UPSC civil

INSIGHTSIAS SIMPLYFYING IAS EXAM PREPARATION

INSTA Tests

37 to 40 (GS)

www.insightsonindia.com

prelims.insightsonindia.com | mains.insightsonindia.com

Telegram: insightsIAStips | FB: insightsonindia | TW: vinaygb | YT: INSIGHTS IAS

BENGALURU | DELHI | HYDERABAD

INSTA 75 Days REVISION PLAN UPSC Prelims 2020

Copyright © by Insights IAS All rights are reserved. No part of this document may be reproduced, stored in a retrieval system or transmitted in any form or by any means, electronic, mechanical, photocopying, recording or otherwise, without prior permission of Insights IAS.

KEY & EXPLANATIONS

Page 2: SIMPLYFYING IAS EXAM PREPARATION - INSIGHTSIAS€¦ · This document is the compilation of 100 questions that are part of InsightsIAS famous INSTA REVISION initiative for UPSC civil

Insta 75 Days Revision Plan for UPSC Civil Services

Prelims – 2020

This document is the compilation of 100 questions that are part of InsightsIAS

famous INSTA REVISION initiative for UPSC civil services Preliminary examination

– 2020 (which has become most anticipated annual affair by lakhs of IAS aspirants

across the country). These questions are carefully framed so as to give aspirants tough

challenge to test their knowledge and at the same time improve skills such as

intelligent guessing, elimination, reasoning, deduction etc – which are much needed

to sail through tough Civil Services Preliminary Examination conducted by UPSC.

These questions are based on this INSTA Revision Plan which is posted on our

website (www.insightsonindia.com). Every year thousands of candidates follow our

revision timetable – which is made for SERIOUS aspirants who would like to intensively

revise everything that’s important before the exam.

Those who would like to take up more tests for even better preparation, can

enroll to Insights IAS Prelims Mock Test Series – 2020

(https://prelims.insightsonindia.com). Every year toppers solve our tests and sail

through UPSC civil services exam. Your support through purchase of our tests will help

us provide FREE content on our website seamlessly.

Wish you all the best!

Team InsightsIAS

Page 3: SIMPLYFYING IAS EXAM PREPARATION - INSIGHTSIAS€¦ · This document is the compilation of 100 questions that are part of InsightsIAS famous INSTA REVISION initiative for UPSC civil

INSTA 75 Days REVISION PLAN for Prelims 2020 - InstaTests

www.insightsonindia.com 1 Insights IAS

DAY – 37 (InstaTest-37)

1. Consider the following statements regarding Dwarf Planet

1. It is a planetary-mass object that is neither a planet nor a natural satellite.

2. Dwarf planets are generally smaller than the planet Mercury.

3. Ceres is the largest dwarf planet.

Which of the statements given above is/are correct?

(a) 1 and 2 only

(b) 1 and 3 only

(c) 2 only

(d) 1, 2 and 3

Solution: A

• A dwarf planet is a planetary-mass object that is neither a planet nor a natural

satellite.

• According to the International Astronomical Union, a dwarf planet is a celestial body

that:

1. Orbits the sun.

2. Has enough mass to assume a nearly round shape.

3. Has not cleared the neighborhood around its orbit.

4. Is not a moon.

• The main distinction between a dwarf planet and a planet is that planets have cleared

the path around the sun while dwarf planets tend to orbit in zones of similar objects

that can cross their path around the sun, such as the asteroid and Kuiper belts.

• Dwarf planets also are generally smaller than the planet Mercury.

• The first five recognized dwarf planets are Ceres, Pluto, Eris, Makemake and Haumea.

• Pluto is the largest dwarf planet.

https://www.universetoday.com/72717/what-is-a-dwarf-planet/

2. Consider the following statements regarding GEMINI device

1. It is a device for dissemination of emergency information and communication on

disaster warnings, Potential Fishing Zones (PFZ) and Ocean States Forecasts (OSF)

to fishermen.

2. It has been developed by ISRO.

Page 4: SIMPLYFYING IAS EXAM PREPARATION - INSIGHTSIAS€¦ · This document is the compilation of 100 questions that are part of InsightsIAS famous INSTA REVISION initiative for UPSC civil

INSTA 75 Days REVISION PLAN for Prelims 2020 - InstaTests

www.insightsonindia.com 2 Insights IAS

3. The GEMINI device receives and transfers the data received from GAGAN

satellite/s to a mobile through Bluetooth communication.

Which of the statements given above is/are correct?

(a) 1 only

(b) 2 and 3 only

(c) 1 and 3 only

(d) 1, 2 and 3

Solution: C

Gagan Enabled Mariner’s Instrument for Navigation and Information (GEMINI) device:

• It is a device that used for seamless and effective dissemination of emergency

information and communication on disaster warnings, Potential Fishing Zones (PFZ)

and Ocean States Forecasts (OSF) to fishermen.

• The device will help to provide information related to disaster warnings when

fishermen move away from the coast beyond 10 to 12 kilometres.

• The GEMINI device receives and transfers the data received from GAGAN satellite/s

to a mobile through Bluetooth communication. A mobile application developed by

INCOIS decodes and displays the information in nine regional languages.

• It has been developed by Indian National Centre for Ocean Information Services

(INCOIS), and Airports Authority of India (AAI).

• It is electronically designed and manufactured by a private industry M/S Acord,

Bangalore under Make in India Program.

3. Consider the following statements regarding CARTOSAT-3

1. It is the third in the series and launched by PSLV-C47

2. The Cartosat satellites are earth observation satellites, used mainly for large-scale

mapping of the Earth through high-resolution cameras.

Which of the statements given above is/are correct?

(a) 1 only

(b) 2 only

(c) Both 1 and 2

(d) Neither 1 nor 2

Page 5: SIMPLYFYING IAS EXAM PREPARATION - INSIGHTSIAS€¦ · This document is the compilation of 100 questions that are part of InsightsIAS famous INSTA REVISION initiative for UPSC civil

INSTA 75 Days REVISION PLAN for Prelims 2020 - InstaTests

www.insightsonindia.com 3 Insights IAS

Solution: B

CARTOSAT-3, which is the ninth in the series, was launched from the second launch pad at

Satish Dhawan Space Centre (SDSC) SHAR, Sriharikota at the scheduled time of 9.28 am. This

was the 74th launch vehicle mission from Sriharikota.

• The Polar Satellite Launch Vehicle PSLV-C47 carried the CARTOSAT-3 satellite along

with the other commercial nano-satellites for the US.

Cartosat Satellites:

• The Cartosat satellites are earth observation satellites, used mainly for large-scale

mapping of the Earth through high-resolution cameras.

• It also helps to detect changes in natural geographical or man-made features. As their

cameras can `look back and forth’ in an angle to generate continuous spot images.

The Earth-observation satellites also include the Resourcesat and RISAT series, the

Oceansat series.

1. The Resourcesat and RISAT series of satellites, for example, provide images and data

that are needed for land and water resources applications.

2. The Oceansat series and the SARAL satellite, meanwhile, produce data on the oceans.

3. The satellites like INSAT 3D, INSAT-VRR or Megha Tropiques study the atmosphere.

What is Cartosat-3?

• Cartosat-3 is a third-generation agile advanced earth observation satellite with high-

resolution imaging capability. Developed by the Indian Space Research Organization

(ISRO), it will replace the IRS series.

• Cartosat-3 has a panchromatic resolution of 0.25 metres making it the imaging

satellite with highest resolution and Mx of 1 metre with a high-quality resolution,

which is a major improvement from the previous payloads in the Cartosat series.

• Cartosat-2 was used to plan and execute military operations such as ‘surgical strikes’

across the Line of Control in 2016 and the operations across Manipur-Myanmar border

in 2015. Cartosat-2 has got resolution of 65 cm.

4. Consider the following statements regarding Dark Matter and Dark Energy

1. Dark energy is a mysterious force that drives the accelerating expansion of the

universe.

2. Dark matter only interacts with the rest of the universe through its gravity.

3. Dark Energy constitutes roughly 27% and Dark matter makes up about 68% of the

universe.

Which of the statement given above is/are correct?

(a) 2 and 3 only

Page 6: SIMPLYFYING IAS EXAM PREPARATION - INSIGHTSIAS€¦ · This document is the compilation of 100 questions that are part of InsightsIAS famous INSTA REVISION initiative for UPSC civil

INSTA 75 Days REVISION PLAN for Prelims 2020 - InstaTests

www.insightsonindia.com 4 Insights IAS

(b) 1 only

(c) 1 and 2 only

(d) 1 and 3 only

Solution: C

• Most of the universe is made up of dark energy, a mysterious force that drives the

accelerating expansion of the universe.

• The next largest ingredient is dark matter, which only interacts with the rest of the

universe through its gravity.

• Normal matter, including all the visible stars, planets and galaxies, makes up less than

5 percent of the total mass of the universe.

• Dark Energy constitutes roughly 68% of the universe. Dark matter makes up about

27%.

Extra Reading:

Dark Matter

• Dark matter is dark: It emits no light and cannot be seen directly, so it cannot be stars

or planets.

• Dark matter is not clouds of normal matter: Normal matter particles are called

baryons. If dark matter were composed of baryons it would be detectable through

reflected light.

• Dark matter is not antimatter: Antimatter annihilates matter on contact, producing

gamma rays. Astronomers do not detect them.

• Dark matter is not black holes: Black holes are gravity lenses that bend light.

Astronomers do not see enough lensing events to account for the amount of dark

matter that must exist

5. Consider the following statements regarding NewSpace India Limited

1. It is a wholly-owned Government of India Company, under the administrative

control of Department of Space (DOS).

2. NSIL is the commercial arm of Indian Space Research Organisation (ISRO).

3. It will be nodal agency for carrying out PSLV production through Indian Industry

under consortium route.

Which of the statements given above is/are correct?

(a) 1 and 2 only

Page 7: SIMPLYFYING IAS EXAM PREPARATION - INSIGHTSIAS€¦ · This document is the compilation of 100 questions that are part of InsightsIAS famous INSTA REVISION initiative for UPSC civil

INSTA 75 Days REVISION PLAN for Prelims 2020 - InstaTests

www.insightsonindia.com 5 Insights IAS

(b) 2 and 3 only

(c) 1 and 3 only

(d) 1, 2 and 3

Solution: D

• In order to commercially exploit the products and services emanating from the Indian

Space Programme, the “NewSpace India Limited (NSIL)” has been incorporated in

March 2019, as a wholly-owned Government of India Company, under the

administrative control of Department of Space (DOS). NSIL currently has an

authorized share capital of ₹ 100 crore and paid-up capital of ₹ 10 crore. NSIL is the

commercial arm of Indian Space Research Organisation (ISRO).

Polar Satellite Launch Vehicle (PSLV) Production

• NewSpace India Limited (NSIL) will be nodal agency for carrying out PSLV production

through Indian Industry under consortium route. The industry consortium will be

responsible for producing, assembling and integrating the launch vehicle, by making

use of the existing ISRO facilities under GOCO model.

6. Consider the following statements regarding Akash Missile

1. This is surface to air anti-aircraft missile

2. The missile is guided by a phased array fire control radar called Rajendra.

3. It has a strike range of 250 km and capability to carry warhead of 60 kilogram.

Which of the statements given above is/are correct?

(a) 1 and 2 only

(b) 2 and 3 only

(c) 1 and 3 only

(d) 1, 2 and 3

Solution: A

Akash Missile:

• Defence Research and Development Organisation (DRDO) has successfully test fired

AKASH-MK-1S missile.

Page 8: SIMPLYFYING IAS EXAM PREPARATION - INSIGHTSIAS€¦ · This document is the compilation of 100 questions that are part of InsightsIAS famous INSTA REVISION initiative for UPSC civil

INSTA 75 Days REVISION PLAN for Prelims 2020 - InstaTests

www.insightsonindia.com 6 Insights IAS

Key facts:

• This is surface to air anti-aircraft missile with a strike range of 25 km and capability

to carry warhead of 60 kilogram.

• It can reach an altitude of 18 km and can be fired from both tracked and wheeled

platforms.

• The missile is guided by a phased array fire control radar called ‘Rajendra’ which is

termed as Battery Level Radar (BLR) with a tracking range of about 60 km.

• The Akash-MK-1S is capable of striking down enemy fighter jets and drones very

effectively and accurately.

• The Akash surface-to-air missile was designed to intercept enemy aircraft and missiles

from a distance of 18 to 30 km.

7. Consider the following statements regarding Gravitational Waves

1. Any object with mass that accelerates produces gravitational waves including

humans, cars and airplanes.

2. Gravitational waves interact very strongly with the matter

3. Albert Einstein predicted the existence of gravitational waves

Select the correct answer using the code given below

(a) 1 and 3 only

(b) 3 only

(c) 2 only

(d) None

Solution: A

• Gravitational waves are ‘ripples’ in the fabric of space-time caused by some of the

most violent and energetic processes in the Universe like colliding black holes, the

collapse of stellar cores (supernovae), coalescing neutron stars or white dwarf stars,

remnants of gravitational radiation created by the birth of the Universe itself.

Sources of G waves

• Any object with mass that accelerates (which in science means changes position at a

variable rate, and includes spinning and orbiting objects) produces gravitational

waves, including humans and cars and airplanes etc.

• But the gravitational waves made by us here on Earth are much too small to detect.

Page 9: SIMPLYFYING IAS EXAM PREPARATION - INSIGHTSIAS€¦ · This document is the compilation of 100 questions that are part of InsightsIAS famous INSTA REVISION initiative for UPSC civil

INSTA 75 Days REVISION PLAN for Prelims 2020 - InstaTests

www.insightsonindia.com 7 Insights IAS

• In fact, it isn’t even remotely possible to build a machine that can spin an object fast

enough to produce a detectible gravitational wave – even the world’s strongest

materials would fly apart at the rotation speeds such a machine would require.

• Since we can’t generate detectable gravitational waves on Earth, the only way to study

them is to look to the places in the Universe where they are generated by nature.

• Gravitational waves interact very weakly with matter (unlike electromagnetic

radiation), they travel through the Universe virtually unimpeded giving us a clear view

of the gravitational-wave Universe.

• Albert Einstein predicted the existence of gravitational waves in 1916 in his general

theory of relativity.

https://www.ligo.caltech.edu/page/what-are-gw

8. Consider the following statements regarding Avangard system

1. It is a new intercontinental strategic system developed by USA.

2. It can fly 27 times the speed of sound and a hypersonic boost-glide missile system.

3. The Avangard missiles have a range of over 6,000 km and weigh approximately

2,000 kg.

Which of the statements given above is/are correct?

(a) 1 and 2 only

(b) 2 and 3 only

(c) 1 and 3 only

(d) 1, 2 and 3

Solution: B

Avangard system:

• Starting from next year, in 2019, a new intercontinental strategic system Avangard

will enter service in the Russian army and the first regiment in the Strategic Missile

Troops will be deployed.

About Avangard system:

• Avangard, also known as “Objekt 4202,” is a hypersonic boost-glide missile system

that is supposed to combine a high-performance ballistic missile with an unmanned

glider vehicle for significant improvements in maneuverability and sustained top

speed.

• It is Russian military’s first Avangard hypersonic intercontinental ballistic missile

(ICBM).

Page 10: SIMPLYFYING IAS EXAM PREPARATION - INSIGHTSIAS€¦ · This document is the compilation of 100 questions that are part of InsightsIAS famous INSTA REVISION initiative for UPSC civil

INSTA 75 Days REVISION PLAN for Prelims 2020 - InstaTests

www.insightsonindia.com 8 Insights IAS

• It can fly 27 times the speed of sound.

• The missile system is a reentry body carried atop an existing ballistic missile, which

has the capability to manoeuvre. It’s manoeuvring capability makes it difficult to

predict its trajectory and gives it the ability to protect itself from the air and ballistic

missile defences by delivering nuclear warheads to targets

• The Avangard missiles have a range of over 6,000 km, weigh approximately 2,000 kg

and can withstand temperatures of over 2000-degree celsius.

9. Consider the following statements regarding ICGS Varah

1. It is a new stealth frigate launched by the Indian Coast Guard (ICG).

2. It is designed and built indigenously by Larsen & Toubro (L&T).

Which of the statements given above is/are correct?

(a) 1 only

(b) 2 only

(c) Both 1 and 2

(d) Neither 1 nor 2

Solution: B

ICGS Varaha:

• It is a new Offshore patrol vessel (OPV) launched by the Indian Coast Guard (ICG). It

is fourth in the series of 98 M OPVs designed and built indigenously by Larsen &

Toubro (L&T).

Key features:

• It is fitted with advanced technology navigation and communication equipment,

sensor and machinery. Its weaponry includes one 30 mm and two 12.7 mm guns with

fire control system. It has been designed to attain maximum speed of 26 knots and

has endurance of 5,000 nautical miles.

• It also equipped with an Integrated Bridge System (IBS), Automated Power

Management System (APMS), Integrated Platform Management System (IPMS), and

High-Power External Fire Fighting (EFF) system.

Page 11: SIMPLYFYING IAS EXAM PREPARATION - INSIGHTSIAS€¦ · This document is the compilation of 100 questions that are part of InsightsIAS famous INSTA REVISION initiative for UPSC civil

INSTA 75 Days REVISION PLAN for Prelims 2020 - InstaTests

www.insightsonindia.com 9 Insights IAS

10. Which of the following diseases/disorders is/are genetic disorders?

1. Down Syndrome

2. Sickle Cell Anemia

3. Hemophilia

Select the correct answer using the code given below

(a) 2 only

(b) 1 and 2 only

(c) 1, 2 and 3

(d) None

Solution: C

• A genetic disorder is a disease that is caused by a change, or mutation, in an

individual’s DNA sequence.

https://www.yourgenome.org/facts/what-is-a-genetic-disorder

11. Consider the following statements regarding measles

1. Measles is a highly contagious viral disease.

2. India is the fifth country in WHO’s Southeast Asia region to eliminate measles.

Page 12: SIMPLYFYING IAS EXAM PREPARATION - INSIGHTSIAS€¦ · This document is the compilation of 100 questions that are part of InsightsIAS famous INSTA REVISION initiative for UPSC civil

INSTA 75 Days REVISION PLAN for Prelims 2020 - InstaTests

www.insightsonindia.com 10 Insights IAS

3. It remains an important cause of death among young children globally, despite

the availability of a vaccine.

Which of the statements given above is/are correct?

(a) 1 and 2 only

(b) 2 and 3 only

(c) 1 and 3 only

(d) 1, 2 and 3

Solution: C

• Sri Lanka has made health history after spending three years free of any new measles

cases and the World Health Organization (WHO) has declared that the deadly

childhood infection has been eliminated in the island nation.

• Sri Lanka is the fifth country in WHO’s Southeast Asia region to eliminate measles.

The other four countries are Bhutan, Maldives, DPR Korea and Timor-Leste.

About Measles:

• Measles is a highly contagious viral disease. It remains an important cause of death

among young children globally, despite the availability of a safe and effective vaccine.

• Spread: Measles is transmitted via droplets from the nose, mouth or throat of

infected persons.

• Initial symptoms, which usually appear 10–12 days after infection, include high fever,

a runny nose, bloodshot eyes, and tiny white spots on the inside of the mouth. Several

days later, a rash develops, starting on the face and upper neck and gradually

spreading downwards.

• Vulnerability: Severe measles is more likely among poorly nourished young children,

especially those with insufficient vitamin A, or whose immune systems have been

weakened by HIV/AIDS or other diseases.

• Prevention: Routine measles vaccination for children, combined with mass

immunization campaigns in countries with low routine coverage, are key public health

strategies to reduce global measles deaths.

• Preventive efforts: Under the Global Vaccine Action Plan, measles and rubella are

targeted for elimination in five WHO Regions by 2020. WHO is the lead technical

agency responsible for coordination of immunization and surveillance activities

supporting all countries to achieve this goal.

• India, along with ten other WHO South East Asia Region member countries, have

resolved to eliminate measles and control rubella/congenital rubella syndrome

(CRS) by 2020. In this direction, Ministry of Health & Family Welfare has initiated

measles-rubella (MR) vaccination campaign in the age group of 9 months to less than

Page 13: SIMPLYFYING IAS EXAM PREPARATION - INSIGHTSIAS€¦ · This document is the compilation of 100 questions that are part of InsightsIAS famous INSTA REVISION initiative for UPSC civil

INSTA 75 Days REVISION PLAN for Prelims 2020 - InstaTests

www.insightsonindia.com 11 Insights IAS

15 years in a phased manner across the nation. The campaign aims to cover

approximately 41 crore children.

• The Measles-Rubella campaign is a part of global efforts to reduce illness and deaths

due to measles and rubella/CRS in the country. Measles immunization directly

contributes to the reduction of under-five child mortality, and in combination with

rubella vaccine, it will control rubella and prevent CRS.

12. Consider the following statements regarding BIO International Convention

1. It is the largest global event for the biotechnology industry and attracts the

biggest names in biotech, offers key networking and partnering opportunities.

2. It is hosted by the UN Environment Programme.

Which of the statements given above is/are correct?

(a) 1 only

(b) 2 only

(c) Both 1 and 2

(d) Neither 1 nor 2

Solution: A

• The BIO International Convention is the largest global event for the biotechnology

industry and attracts the biggest names in biotech, offers key networking and

partnering opportunities, and provides insights and inspiration on the major trends

affecting the industry. The first BIO International Convention was held in 1993 and

attracted approximately 1,400 attendees.

• The event features keynotes and sessions from key policymakers, scientists, CEOs, and

celebrities. The Convention also features the BIO Business Forum (One-on-One

Partnering), hundreds of sessions covering biotech trends, policy issues and

technological innovations, and the world’s largest biotechnology exhibition – the BIO

Exhibition.

• The BIO International Convention is hosted by the Biotechnology Innovation

Organization (BIO). BIO represents more than 1,100 biotechnology companies,

academic institutions, state biotechnology centers and related organizations across

the United States and in more than 30 other nations. BIO members are involved in the

research and development of innovative healthcare, agricultural, industrial and

environmental biotechnology products.

Page 14: SIMPLYFYING IAS EXAM PREPARATION - INSIGHTSIAS€¦ · This document is the compilation of 100 questions that are part of InsightsIAS famous INSTA REVISION initiative for UPSC civil

INSTA 75 Days REVISION PLAN for Prelims 2020 - InstaTests

www.insightsonindia.com 12 Insights IAS

13. Which of the following diseases is/are neglected tropical diseases

1. Rabies

2. Dengue

3. Sleeping sickness

Select the correct answer using the code given below

(a) 1 and 3 only

(b) 2 only

(c) 1 and 2 only

(d) 1, 2 and 3

Solution: D

• Neglected tropical diseases (NTDs)– a diverse group of communicable diseases that

prevail in tropical and subtropical conditions in 149 countries – affect more than one

billion people and cost developing economies billions of dollars every year.

• Populations living in poverty, without adequate sanitation and in close contact with

infectious vectors and domestic animals and livestock are those worst affected.

14. Consider the following statements regarding IndiGen Genome project

1. IndiGen Genome project is conducted by CSIR.

2. It aims to undertake whole genome sequencing of thousands of individuals

representing diverse ethnic groups from India.

3. It will help in development of preventive medicine with faster and efficient

diagnosis of rare genetic diseases.

Which of the statements given above is/are correct?

Page 15: SIMPLYFYING IAS EXAM PREPARATION - INSIGHTSIAS€¦ · This document is the compilation of 100 questions that are part of InsightsIAS famous INSTA REVISION initiative for UPSC civil

INSTA 75 Days REVISION PLAN for Prelims 2020 - InstaTests

www.insightsonindia.com 13 Insights IAS

(a) 1 and 2 only

(b) 2 and 3 only

(c) 1 and 3 only

(d) 1, 2 and 3

Solution: D

IndiGen Genome project

• Details of the IndiGen Genome project, conducted by CSIR, were recently announced.

• The initiative was implemented by the CSIR-Institute of Genomics and Integrative

Biology (IGIB), Delhi and CSIR-Centre for Cellular and Molecular Biology (CCMB),

Hyderabad.

Significance, outcomes and benefits of the project:

• The outcomes of the IndiGen will have applications in a number of areas including

predictive and preventive medicine with faster and efficient diagnosis of rare genetic

diseases.

• The data will be important for building the knowhow, baseline data and indigenous

capacity in the emerging area of Precision Medicine.

About Genomics for Public Health in India (IndiGen) programme:

• IndiGen programme aims to undertake whole genome sequencing of thousands of

individuals representing diverse ethnic groups from India.

• The objective is to enable genetic epidemiology and develop public health

technologies applications using population genome data.

Need for genome sequencing:

• Ever since the human genome was first sequenced in 2003, it opened a fresh

perspective on the link between disease and the unique genetic make-up of each

individual.

• Nearly 10,000 diseases — including cystic fibrosis, thalassemia — are known to be the

result of a single gene malfunctioning.

• While genes may render some insensitive to certain drugs, genome sequencing has

shown that cancer too can be understood from the viewpoint of genetics, rather than

being seen as a disease of certain organs.

Page 16: SIMPLYFYING IAS EXAM PREPARATION - INSIGHTSIAS€¦ · This document is the compilation of 100 questions that are part of InsightsIAS famous INSTA REVISION initiative for UPSC civil

INSTA 75 Days REVISION PLAN for Prelims 2020 - InstaTests

www.insightsonindia.com 14 Insights IAS

15. Consider the following statements regarding SDG Gender Index

1. It is developed by UNDP.

2. A score of 100 reflects the achievement of gender equality in relation to the

targets set for each indicator.

3. India is ranked 95th among 129 countries.

Which of the statements given above is/are correct?

(a) 1 and 2 only

(b) 2 and 3 only

(c) 1 and 3 only

(d) 1, 2 and 3

Solution: B

SDG Gender Index

• Developed by Equal Measures 2030, a joint effort of regional and global organisations

including African Women’s Development and Communication Network, Asian-

Pacific Resource and Research Centre for Women, Bill and Melinda Gates

Foundation etc.

• It accounts for 14 out of 17 SDGs (sustainable development goals) that cover aspects

such as poverty, health, education, literacy, political representation and equality at

the workplace.

• A score of 100 reflects the achievement of gender equality in relation to the targets

set for each indicator. It means, for example, that 100% of girls complete secondary

education, or that there is around 50-50 parity for women and men in Parliament. A

score of 50 signifies that a country is about halfway to meeting a goal.

Key findings- India specific:

• India is ranked 95th among 129 countries.

• India’s highest goal scores are on health (79.9), hunger & nutrition (76.2), and energy

(71.8).

• Its lowest goal scores are on partnerships (18.3, in the bottom 10 countries

worldwide), industry, infrastructure and innovation (38.1), and climate (43.4).

• On indicators that define such goals, India scored 95.3 on the percentage of female

students enrolled in primary education who are overage.

• Some of India’s lowest scores on indicators include the proportion of seats held by

women in national parliaments. On seats held by women in the Supreme Court (4%),

India has a score of 18.2.

Page 17: SIMPLYFYING IAS EXAM PREPARATION - INSIGHTSIAS€¦ · This document is the compilation of 100 questions that are part of InsightsIAS famous INSTA REVISION initiative for UPSC civil

INSTA 75 Days REVISION PLAN for Prelims 2020 - InstaTests

www.insightsonindia.com 15 Insights IAS

Gender-based violence

• On gender-based violence, indicators include proportion of women aged 20-24 years

who were married or in a union before age 18 (27.3%).

16. Consider the following statements regarding Foreign Contribution (Regulation) Act,

2010

1. It regulates the receipt and usage of foreign contribution by non-governmental

organizations in India.

2. It is implemented by the Ministry of Home Affairs.

3. If someone violates the FCRA act, he can be sent to jail for up to 5 years.

Which of the statements given above is/are correct?

(a) 1 and 3 only

(b) 2 and 3 only

(c) 1 and 2 only

(d) 1, 2 and 3

Solution: D

• The Foreign Contribution (Regulation) Act, 2010 and rules framed under it (the

“FCRA” or “Act”) regulate the receipt and usage of foreign contribution by non-

governmental organisations (“NGOs”) in India. Since the Act is internal security

legislation, despite being a law related to financial legislation, it falls into the purview

of Home Ministry and not the Reserve Bank of India (RBI).

Scope and objective of FCRA:

• The intent of the Act is to prevent use of foreign contribution or foreign hospitality for

any activity detrimental to the national interest.

• It has a very wide scope and is applicable to a natural person, body corporate, all other

types of Indian entities (whether incorporated or not) as well as NRIs and overseas

branches/subsidiaries of Indian companies and other entities formed or registered in

India. It is implemented by the Ministry of Home Affairs, Government of India.

• If someone violates the FCRA act, he can be sent to jail for up to 5 years.

• The Acts ensures that the recipients of foreign contributions adhere to the stated

purpose for which such contribution has been obtained.

• Under the Act, organisations require to register themselves every five years

Page 18: SIMPLYFYING IAS EXAM PREPARATION - INSIGHTSIAS€¦ · This document is the compilation of 100 questions that are part of InsightsIAS famous INSTA REVISION initiative for UPSC civil

INSTA 75 Days REVISION PLAN for Prelims 2020 - InstaTests

www.insightsonindia.com 16 Insights IAS

17. Consider the following statements regarding National Defence Fund (NDF)

1. It was set up to take charge of the Voluntary donations in cash and kind received

for promotion of the national Defence effort.

2. The fund is administered by an Executive Committee with the Defence Minister

as the Chairperson.

Which of the statements given above is/are correct?

(a) 1 only

(b) 2 only

(c) Both 1 and 2

(d) Neither 1 nor 2

Solution: A

National Defence Fund (NDF):

• A major change has been brought about in the ‘Prime Minister’s Scholarship Scheme’

under the National Defence Fund.

Key changes:

• Scholarships increase from ₹2,000 to ₹2,500 a month for boys and from ₹2,250 to

₹3,000 a month for girls.

• The ambit of the scheme was widened to cover the wards of State police officials

martyred in terror or Naxal attacks.

• new scholarships are given to 5,500 wards of armed forces personnel, 2,000 wards of

paramilitary forces personnel and 150 wards of forces under the Railways Ministry

every year.

About National Defence Fund (NDF):

• The National Defence Fund (NDF) was set up in 1962 to take charge of the Voluntary

donations in cash and kind received for promotion of the national Defence effort, and

to decide on their utilisation.

• Currently the fund is being used for the welfare of the members of the Armed Forces,

Para Military forces and Railway Protection Force, and their dependents.

• The fund is administered by an Executive Committee with the Prime Minister as the

Chairperson and the Defence, Finance and Home Ministers as Members.

• Under National Defence Fund major scheme of ‘Prime Minister’s Scholarship Scheme

(PMSS)’ is being implemented to encourage technical and post-graduate education

for the widows and wards of the deceased/ex-service personnel.

Page 19: SIMPLYFYING IAS EXAM PREPARATION - INSIGHTSIAS€¦ · This document is the compilation of 100 questions that are part of InsightsIAS famous INSTA REVISION initiative for UPSC civil

INSTA 75 Days REVISION PLAN for Prelims 2020 - InstaTests

www.insightsonindia.com 17 Insights IAS

• Scholarships are available for education at technical institutions (medical, dental,

veterinary, engineering, MBA, MCA and other equivalent technical professions with

suitable AICTE/UGC approval).

• The National Defence Fund accepts online voluntary contributions through the

website.

18. Consider the following statements regarding AWaRe tool

1. It has been developed by Indian Council of Medical Research (ICMR).

2. It is an online tool aimed at guiding policy-makers and health workers to use

antibiotics safely and more effectively.

Which of the statements given above is/are correct?

(a) 1 only

(b) 2 only

(c) Both 1 and 2

(d) Neither 1 nor 2

Solution: B

AWaRe- a WHO tool for safer use of antibiotics

• WHO launches tool for safer use of antibiotics, curb resistance- AWaRe.

About AWaRe:

• It is an online tool aimed at guiding policy-makers and health workers to use

antibiotics safely and more effectively.

The tool, known as ‘AWaRe’, classifies antibiotics into three groups:

1. Access — antibiotics used to treat the most common and serious infections.

2. Watch — antibiotics available at all times in the healthcare system.

3. Reserve — antibiotics to be used sparingly or preserved and used only as a last resort.

Concerns:

• Antibiotic resistance is already one of the biggest health risks and is estimated to kill

50 million by 2050 worldwide.

• The threat continues to escalate globally because more than 50 per cent of antibiotics

in many countries are used inappropriately such as for treatment of viruses when they

only treat bacterial infections or use of the wrong (broader spectrum) antibiotic.

Page 20: SIMPLYFYING IAS EXAM PREPARATION - INSIGHTSIAS€¦ · This document is the compilation of 100 questions that are part of InsightsIAS famous INSTA REVISION initiative for UPSC civil

INSTA 75 Days REVISION PLAN for Prelims 2020 - InstaTests

www.insightsonindia.com 18 Insights IAS

• Besides, reduced access to effective and appropriate antibiotics in many low- and

middle-income countries contributes to childhood deaths and lack of funding and

implementation of national plans to tackle antimicrobial resistance.

Need of the hour:

• With the emergence of infections that are untreatable by all classes of antibiotics,

antimicrobial resistance has become “an invisible pandemic”.

• In the absence of development of new drugs, “we must safeguard these precious last-

line antibiotics to ensure we can still treat and prevent serious infections”.

19. Consider the following statements regarding Food Safety and Standards Authority of

India (FSSAI):

1. Ministry of Food Processing Industries is the Administrative Ministry for the

implementation of FSSAI.

2. It has been established under the Prevention of Food Adulteration Act, 1954.

3. It is responsible for protecting and promoting public health through the regulation

and supervision of food safety.

Which of the statements given above is/are correct?

(a) 1 and 3 only

(b) 2 and 3 only

(c) 3 only

(d) 1, 2 and 3

Solution: C

Food Safety and Standards Authority of India (FSSAI)

• The Food Safety and Standards Authority of India (FSSAI) has been established under

Food Safety and Standards Act, 2006 which consolidates various acts & orders that

have hitherto handled food related issues in various Ministries and Departments.

• It was created for laying down science based standards for articles of food and to

regulate their manufacture, storage, distribution, sale and import to ensure

availability of safe and wholesome food for human consumption.

• Ministry of Health & Family Welfare, Government of India is the Administrative

Ministry for the implementation of FSSAI.

• The Chairperson and Chief Executive Officer of Food Safety and Standards Authority

of India (FSSAI) are appointed by Government of India.

• The Chairperson is in the rank of Secretary to Government of India.

Page 21: SIMPLYFYING IAS EXAM PREPARATION - INSIGHTSIAS€¦ · This document is the compilation of 100 questions that are part of InsightsIAS famous INSTA REVISION initiative for UPSC civil

INSTA 75 Days REVISION PLAN for Prelims 2020 - InstaTests

www.insightsonindia.com 19 Insights IAS

• FSSAI is responsible for protecting and promoting public health through the regulation

and supervision of food safety.

20. Recently a tortois, named Impressive tortoise was discovered in

(a) Kerala

(b) Andaman and Nicobar Islands

(c) Arunachal Pradesh

(d) Orissa

Solution: C

Impressive tortoise from Arunachal Pradesh:

• A tortoise considered beautiful enough to be named ‘impressed’ has been discovered

in Arunachal Pradesh.

• This is the first record of the tortoise in India, taking the count to five and the non-

marine chelonian count to 29. Chelonian is an order of reptile that includes turtles,

terrapins and tortoises.

• India was known to be the home of only the Asian Forest Tortoise (Manouria emys)

until the discovery of the Impressed Tortoise.

• The Asian Forest Tortoise, the largest in mainland Asia, is found only in the northeast,

as are 20 of the other 28 species of chelonians.

21. Consider the following statements regarding the Indian Council of Medical Research

(ICMR):

1. It is the apex body in India for the formulation, coordination and promotion of

biomedical research.

2. It is funded by the Government of India through the Department of Health

Research, Ministry of Health & Family Welfare

3. The Governing Body of the Council is presided over by the Union Health Minister.

Which of the statements given above is/are correct?

(a) 1 and 2 only

(b) 2 and 3 only

(c) 1 and 3 only

(d) 1, 2 and 3

Page 22: SIMPLYFYING IAS EXAM PREPARATION - INSIGHTSIAS€¦ · This document is the compilation of 100 questions that are part of InsightsIAS famous INSTA REVISION initiative for UPSC civil

INSTA 75 Days REVISION PLAN for Prelims 2020 - InstaTests

www.insightsonindia.com 20 Insights IAS

Solution: D

• The Indian Council of Medical Research (ICMR), New Delhi, the apex body in India for

the formulation, coordination and promotion of biomedical research, is one of the

oldest medical research bodies in the world.

• It was established in 1911 as Indian Research Fund Association (IRFA).

• IRFA was redesignated as the Indian Council of Medical Research (with Dr. C.G. Pandit

as its first Director).

• The ICMR is funded by the Government of India through the Department of Health

Research, Ministry of Health & Family Welfare.

• The Council’s research priorities coincide with the National health priorities such as

control and management of communicable diseases, fertility control, maternal and

child health, control of nutritional disorders, developing alternative strategies for

health care delivery, containment within safety limits of environmental and

occupational health problems; research on major non-communicable diseases like

cancer, cardiovascular diseases, blindness, diabetes and other metabolic and

haematological disorders; mental health research and drug research (including

traditional remedies). All these efforts are undertaken with a view to reduce the total

burden of disease and to promote health and well-being of the population.

• The Governing Body of the Council is presided over by the Union Health Minister. It is

assisted in scientific and technical matters by a Scientific Advisory Board comprising

eminent experts in different biomedical disciplines. The Board, in its turn, is assisted

by a series of Scientific Advisory Groups, Scientific Advidsory Committees, Expert

Groups, Task Forces, Steering Committees etc. which evaluate and moniter different

research activities of the Council.

• The Council promotes biomedical research in the country through intramural as well

as extramural research. Over the decades, the base of extramural research and also

its strategies have been expanded by the Council.

• It is important for the exam as it is playing pivotal role in testing of Covid-19 pandemic.

https://icmr.nic.in/content/about-us#

22. Consider the following statements regarding report: State of India’s Environment 2019

1. It is an annual quantified statement of environmental statistics and analysis put

together by Down To Earth magazine.

2. Air pollution is responsible for 12.5 per cent of all deaths in India.

Which of the statements given above is/are correct?

(a) 1 only

(b) 2 only

(c) Both 1 and 2

Page 23: SIMPLYFYING IAS EXAM PREPARATION - INSIGHTSIAS€¦ · This document is the compilation of 100 questions that are part of InsightsIAS famous INSTA REVISION initiative for UPSC civil

INSTA 75 Days REVISION PLAN for Prelims 2020 - InstaTests

www.insightsonindia.com 21 Insights IAS

(d) Neither 1 nor 2

Solution: C

State of India’s Environment 2019

• The State of India’s Environment 2019 in Figures is an exclusive data-driven analysis

of major developmental and environmental sectors. SoE 2019 in Figures is an annual

quantified statement of environmental statistics and analysis put together by Down

To Earth magazine, which Centre for Science and Environment (CSE) helps publish.

• Usefulness: The datasets can be used by the media to investigate compelling stories,

ask better questions to policymakers to drive them to come up with better policies for

sustainable development agenda.

Key findings:

State of air – Air pollution is responsible for 12.5 per cent of all deaths in India. Its impact on

children is equally worrying. Over 100,000 children below the age of five die due to bad air in

the country. While India was one of the first countries to pledge the phasing out of non-

electric vehicles, its national scheme to promote the sale of e-vehicles is yet to pick up. Against

the target of 15-16 million e-vehicles by 2020, the county had 0.28 million vehicles till May

2019.

State of development – Climate change poses the biggest economic threat in the world today

and features prominently in the UN Sustainable Development Goals 2030. With just 10 years

to go, India is yet to identify indicators to track its climate change preparedness. Of the 13

SDGs the country is tracking, indicators exist for only a handful of the targets.

State of water – Both surface and groundwater in the country are under stress. 86 water

bodies are critically polluted. The bulk of the polluted water bodies are in Karnataka,

Telangana and Kerala. One of the reasons is the substantial increase (136 per cent) in the

number of grossly polluting industries between 2011 and 2018. Groundwater is also reeling

under overexploitation, which is running 94.5 per cent of all minor irrigation schemes in the

country. There has been an unsustainable increase in the number of deep tubewells that has

gone up by 80 per cent between 2006-07 and 2013-14.

State of land and agriculture – India’s farm sector is under duress. While the input costs for

major crops are rising, the average farmland size is shrinking. Even the share of the insured

cropped area stands at a dismal 26 per cent.

State of Health – India’s rural health infrastructure is ailing. There is a 35 per cent shortfall in

the number of 24×7 public health centres, where 26 per cent of the positions for medical

officers are lying vacant. In fact, Kerala does not have a single 24×7 public health centre.

Another worrying trend is that the number of new doctors qualifying every year in the country

has decreased by 60 per cent between 2013 and 2017. The country also shares the world’s

Page 24: SIMPLYFYING IAS EXAM PREPARATION - INSIGHTSIAS€¦ · This document is the compilation of 100 questions that are part of InsightsIAS famous INSTA REVISION initiative for UPSC civil

INSTA 75 Days REVISION PLAN for Prelims 2020 - InstaTests

www.insightsonindia.com 22 Insights IAS

largest absolute burden of at least 11 major neglected tropical diseases, which includes

diseases like dengue.

State of cities – By 2050, India is projected to add 416 million urban dwellers to the world’s

urban population and will be home to about 58 per cent of the total global population.

Keeping this in mind, India in 2015-16 announced its ambitious plan of creating 100 smart

cities. Four years later, only 21 per cent of the allocated funds for the smart cities have been

spent. In the meanwhile, most urban cities have a sizeable population living in slums, which

are unfit for habitation. India has 2,613 towns with slums. Of them, 57 per cent are in Tamil

Nadu, Madhya Pradesh, Uttar Pradesh, Karnataka and Maharashtra.

State of waste – The burden of solid waste is becoming unmanageable. In fact, 79 major

protests against unsanitary landfills and dump yards have been recorded in 22 states in the

past three years. Maharashtra, which registered 16 major protests, leaves 43 per cent of its

waste unprocessed. While India claims to process 96 per cent of its biomedical waste, eight

states and UTs have defaulting hospitals. The country has also recorded a 56 per cent increase

in the number of hazardous-waste generating industries between 2009 and 2016-17. At the

same time, most of these industries are not properly maintaining their waste inventory, as

mandated by the law.

State of energy – India’s natural gas and hydro-based power plants are in shambles. Gas-

based plants are running at 24 per cent of their capacity due to the acute shortage of domestic

natural gas. Hydropower projects, on the other hand, are running at just 19 per cent of their

capacity and their share in total installed capacity has consistently declined since 1962. The

country’s progress in renewable energy in 2018-19 has also been dismal. In wind, the country

met only 6.3 per cent of the target this year. In solar, it met 5.86 per cent.

State of climate – There has been a 22 per cent increase in India’s greenhouse gas (GHG)

emissions between 2010 and 2014. This has been fuelled by the energy sector, which is

responsible for 73 per cent of the total GHG emissions. Besides, India phased out ozone

depleting substances such as chlorofluorocarbon by 2011, it shifted to substances such as

hydrochlorofluorocarbon, which have high globalwarming potential. India continues to bear

the brunt of extreme weather events. In 2018, 11 states recorded major extreme weather

events that claimed 1,425 lives.

State of forests – India has recently shifted to a powerful forest fire monitoring and alert

system, SNPP-VIIRS, which can capture forest fires with better accuracy and precision. In April

2019, the new technology recorded 69,523 forest fires, which was 9.5 times more than that

recorded by the earlier technology.

State of wildlife – 37 species were poached or seized in 2018. Of these, 13, including lion,

marked an increase over the last year; 161 wild animals were also killed due to road and train

accidents

State of employment – India has witnessed a 1.9 times increase in the unemployment rate in

the past two years. This has especially affected the youth and the educated. Unemployment

Page 25: SIMPLYFYING IAS EXAM PREPARATION - INSIGHTSIAS€¦ · This document is the compilation of 100 questions that are part of InsightsIAS famous INSTA REVISION initiative for UPSC civil

INSTA 75 Days REVISION PLAN for Prelims 2020 - InstaTests

www.insightsonindia.com 23 Insights IAS

rate among people with at least a graduate degree was 13.17 per cent in September-

December 2018, up from 10.39 per cent in May-August 2017.

23. Consider the following statements regarding the Himalayan Ibex:

1. Recently, study by the Zoological Survey of India (ZSI) has proved that Himalayan

Ibex is a subspecies of the Siberian Ibex

2. It is endemic to India

3. It is distributed mainly in the trans-Himalayan ranges of the Union Territories of

Ladakh and Jammu and Kashmir and Himachal Pradesh

Which of the statements given above is/are correct?

(a) 2 only

(b) 1 and 3 only

(c) 3 only

(d) 1 and 2 only

Solution: C

• A recent study by scientists of the Zoological Survey of India (ZSI) has proved that

Himalayan Ibex, distributed in the trans-Himalayan ranges of Jammu and Kashmir,

Ladakh and Himachal Pradesh, is a distinct species from the Siberian Ibex.

• The paper, ‘Genetic evidence for allopatric speciation of the Siberian Ibex (Capra

sibirica) in India,’ has recently been published in Endangered Species Research, an

international peer-reviewed journal.

• They are found in the western Himalaya in Pakistan and India, usually at elevations of

3800m and higher.

• Siberian Ibex is a species of wild goat and is distributed in diverse habitats, ranging

from cold deserts, rocky outcrops, steep terrain, high-land flats and mountain ridges

to low mountains and foothills.

• From Mongolia, its distribution extends towards Altai, Hangai, Gobi-Altai, the Hurukh

mountain ranges as well as Sayan Mountains near Russia and scattered populations

in the small mountains of Trans-Altai Gobi.

• In Asia, Ibex is distributed in the Montane habitats, ranging in elevations from 500 m

to 6,700 m in countries like India, Kazakhstan, Tajikistan, Mongolia, Pakistan,

Southern Siberia and China. In India, the Ibex is distributed mainly in the trans-

Himalayan ranges of the Union Territories of Ladakh and Jammu and Kashmir and

Himachal Pradesh up to the river Sutlej.

Page 26: SIMPLYFYING IAS EXAM PREPARATION - INSIGHTSIAS€¦ · This document is the compilation of 100 questions that are part of InsightsIAS famous INSTA REVISION initiative for UPSC civil

INSTA 75 Days REVISION PLAN for Prelims 2020 - InstaTests

www.insightsonindia.com 24 Insights IAS

https://www.thehindu.com/sci-tech/energy-and-environment/himalayan-ibex-a-distinct-

species/article31201581.ece

24. Consider the following statements regarding Renewable Energy Management Centres

(REMC):

1. They are equipped with artificial intelligence-based Renewable Energy forecasting

and scheduling tools.

2. REMCs are being managed by the Power Grid Corporation of India Limited

(POWERGRID) at the regional and national level

3. Union Government had mandated the Power System Operation Corporation of

India Ltd (POSOCO) as the implementing agency for setting up of REMCs.

Which of the statements given above is/are correct?

(a) 1 only

(b) 2 only

(c) 1 and 3 only

(d) 1, 2 and 3

Solution: A

• The Government of India’s target of 175 GW RE Capacity by 2022 driving accelerated

RE penetration poses challenges to the grid management due to intermittent and

variable nature of RE generation. The renewable energy management centers are

equipped with Artificial Intelligence based RE forecasting and scheduling tools and

provide greater visualization and enhanced situational awareness to the grid

operators. The Renewable Energy Management Centers (REMCs) are co-located with

the State Load DispatchCenters (SLDCs) in Tamil Nadu, Karnataka, Andhra Pradesh,

Maharashtra, Madhya Pradesh, Gujarat & Rajasthan and in RLDCs at Bengaluru,

Mumbai and New Delhi and at the NLDC. Presently, 55 GW of Renewable (Solar and

Wind) is being monitored through the eleven REMCs.

• Government of India had approved the implementation of the REMCs as a Central

Scheme and had mandated POWERGRID, a Maharatna CPSE under Ministry of Power

as Implementing Agency.

• These REMCs are being managed by Power System Operation Corporation of India

Ltd. (POSOCO) at Regional and National level and at State level by State Load Despatch

Centres (SLDCs).

https://m.economictimes.com/small-biz/productline/power-generation/power-minister-

dedicates-11-renewable-energy-management-centres-to-nation/articleshow/74438499.cms

Page 27: SIMPLYFYING IAS EXAM PREPARATION - INSIGHTSIAS€¦ · This document is the compilation of 100 questions that are part of InsightsIAS famous INSTA REVISION initiative for UPSC civil

INSTA 75 Days REVISION PLAN for Prelims 2020 - InstaTests

www.insightsonindia.com 25 Insights IAS

https://pib.gov.in/newsite/PrintRelease.aspx?relid=199638

25. Consider the following statements regarding the Geo Imaging Satellite (GISAT-1):

1. It is the first state-of-the-art agile Earth observation satellite which will be placed

in a Geosynchronous Transfer Orbit.

2. Gisat-1 will facilitate near real time observation of the Indian subcontinent, under

cloud free condition, at frequent intervals

Which of the statements given above is/are correct?

(a) 1 only

(b) 2 only

(c) Both 1 and 2

(d) Neither 1 nor 2

Solution: C

• The launch of GISAT-1 onboard GSLV-F10, planned for March 05, 2020, is postponed

due to technical reasons.

• Geosynchronous Satellite Launch Vehicle (GSLV-F10) will launch Geo Imaging

Satellite (GISAT-1) from the Second Launch Pad of Satish Dhawan Space Centre (SDSC)

SHAR, Sriharikota.

• Weighing about 2268 kg, GISAT-1 is the first state-of-the-art agile Earth observation

satellite which will be placed in a Geosynchronous Transfer Orbit by GSLV-F10.

Subsequently, the satellite will reach the final geostationary orbit using its onboard

propulsion system.

• A 4 metre diameter Ogive shaped payload fairing is being flown for the first time in

this GSLV flight. This is the fourteenth flight of the GSLV.

• Operating from geostationary orbit, GISAT-1 will facilitate near real time observation

of the Indian sub-continent, under cloud free condition, at frequent intervals.

https://www.isro.gov.in/launcher/gslv-f10-gisat-1

DAY – 38 (InstaTest-38)

26. Consider the following statements regarding United Nations Convention to Combat

Desertification (UNCCD)

1. It is legally binding international agreement

Page 28: SIMPLYFYING IAS EXAM PREPARATION - INSIGHTSIAS€¦ · This document is the compilation of 100 questions that are part of InsightsIAS famous INSTA REVISION initiative for UPSC civil

INSTA 75 Days REVISION PLAN for Prelims 2020 - InstaTests

www.insightsonindia.com 26 Insights IAS

2. It is the only convention stemming from a direct recommendation of the Rio

Conference’s Agenda 21.

3. The Ministry of Environment, Forest and Climate Change is the nodal Ministry for

this Convention.

Which of the statements given above is/are correct?

(a) 1 only

(b) 2 and 3 only

(c) 1 and 3 only

(d) 1, 2 and 3

Solution: D

United Nations Convention to Combat Desertification (UNCCD)

• Established in 1994.

• It is the sole legally binding international agreement linking environment and

development to sustainable land management.

• It is the only convention stemming from a direct recommendation of the Rio

Conference’s Agenda 21.

• To help publicise the Convention, 2006 was declared “International Year of Deserts

and Desertification”.

• Focus areas: The Convention addresses specifically the arid, semi-arid and dry sub-

humid areas, known as the drylands, where some of the most vulnerable ecosystems

and peoples can be found.

• Aim: Its 197 Parties aim, through partnerships, to implement the Convention and

achieve the Sustainable Development Goals. The end goal is to protect land from over-

use and drought, so it can continue to provide food, water and energy.

• The Ministry of Environment, Forest and Climate Change is the nodal Ministry for this

Convention.

27. Consider the following statements regarding STRIDE scheme

1. It was launched by University Grants Commission of India.

2. It will provide support to research projects that are socially relevant, locally need-

based, nationally important and globally significant.

Which of the statements given above is/are correct?

(a) 1 only

(b) 2 only

Page 29: SIMPLYFYING IAS EXAM PREPARATION - INSIGHTSIAS€¦ · This document is the compilation of 100 questions that are part of InsightsIAS famous INSTA REVISION initiative for UPSC civil

INSTA 75 Days REVISION PLAN for Prelims 2020 - InstaTests

www.insightsonindia.com 27 Insights IAS

(c) Both 1 and 2

(d) Neither 1 nor 2

Solution: C

STRIDE scheme

• UGC announces new Initiative – Scheme for Trans-disciplinary Research for India’s

Developing Economy (STRIDE) to boost research culture in India.

Key features:

• STRIDE will provide support to research projects that are socially relevant, locally

need-based, nationally important and globally significant.

• STRIDE shall support research capacity building as well as basic, applied and

transformational action research that can contribute to national prioritiers with focus

on inclusive human development.

• STRIDE shall support creation, development and integration of new ideas, concepts

and practices for public good and strengthening civil society.

STRIDE Objectives:

• To identify young talent, strengthen research culture, build capacity, promote

innovation and support trans-disciplinary research for India’s developing economy

and national development.

• To fund multi institutional network high-impact research projects in humanities and

human sciences.

Significance:

• STRIDE scheme will strengthen research culture and innovation in colleges and

Universities and help students and faculty to contribute towards India’s developing

economy with help of collaborative research. Focus on Humanities and Human

Sciences will boost quality research on Indian languages and knowledge systems.

28. Consider the following statements regarding Ramanujan Machine

1. Scientists from India Institute of Technology have developed a concept named the

Ramanujan Machine.

2. The purpose of the machine is to come up with conjectures that humans can

analyze, and hopefully prove to be true mathematically.

3. It is not really a machine but an algorithm, and performs a very unconventional

function.

Which of the statements given above is/are correct?

Page 30: SIMPLYFYING IAS EXAM PREPARATION - INSIGHTSIAS€¦ · This document is the compilation of 100 questions that are part of InsightsIAS famous INSTA REVISION initiative for UPSC civil

INSTA 75 Days REVISION PLAN for Prelims 2020 - InstaTests

www.insightsonindia.com 28 Insights IAS

(a) 1 and 2 only

(b) 2 and 3 only

(c) 1 and 3 only

(d) 1, 2 and 3

Solution: B

Ramanujan Machine

• Scientists from Technion — Israel Institute of Technology have developed a concept

they have named the Ramanujan Machine, after the Indian mathematician.

• It is not really a machine but an algorithm, and performs a very unconventional

function.

• The Ramanujan machine is more of a concept than an actual machine—it exists as a

network of computers running algorithms dedicated to finding conjectures about

fundamental constants in the form of continued fractions—these are defined as

fractions of infinite length where the denominator is a certain quantity plus a fraction,

where a latter fraction has a similar denominator, etc.)

• The purpose of the machine is to come up with conjectures (in the form of

mathematical formulas) that humans can analyze, and hopefully prove to be true

mathematically.

Why Ramanujan?

• The algorithm reflects the way Srinivasa Ramanujan worked during his brief life (1887-

1920). With very little formal training, he engaged with the most celebrated

mathematicians of the time, particularly during his stay in England (1914-19), where

he eventually became a Fellow of the Royal Society and earned a research degree from

Cambridge.

• Throughout his life, Ramanujan came up with novel equations and identities —

including equations leading to the value of pi — and it was usually left to formally

trained mathematicians to prove these.

What’s the point?

• Conjectures are a major step in the process of making new discoveries in any branch

of science, particularly mathematics. Equations defining the fundamental

mathematical constants, including pi, are invariably elegant. New conjectures in

mathematics, however, have been scarce and sporadic, the researchers note in their

paper, which is currently on a pre-print server. The idea is to enhance and accelerate

the process of discovery.

Page 31: SIMPLYFYING IAS EXAM PREPARATION - INSIGHTSIAS€¦ · This document is the compilation of 100 questions that are part of InsightsIAS famous INSTA REVISION initiative for UPSC civil

INSTA 75 Days REVISION PLAN for Prelims 2020 - InstaTests

www.insightsonindia.com 29 Insights IAS

How good is it?

• The paper gives examples for previously unknown equations produced by the

algorithm, including for values of the constants pi and e. The Ramanujan Machine

proposed these conjecture formulas by matching numerical values, without providing

proofs. It has to be remembered, however, that these are infinite series, and a human

can only enter a finite number of terms to test the value of the series. The question is,

therefore, whether the series will fail after a point. The researchers feel this is unlikely,

because they tested hundreds of digits.

• Until proven, it remains a conjecture. By the same token, until proven wrong, a

conjecture remains one. It is quite possible that the algorithm will come up with

conjectures that may take years to prove.

29. Consider the following statements regarding Indian Ocean Commission (IOC)

1. It is an intergovernmental organization.

2. Malta and Mauritius are members of the organization.

3. India and Madagascar are observer members of the organization

Which of the statements given above is/are correct?

(a) 1 only

(b) 2 and 3 only

(c) 1 and 3 only

(d) 2 only

Solution: A

• The Indian Ocean Commission (IOC) is an intergovernmental organization which

brings together five member states : the Union of the Comoros, France in respect of

La Réunion, Madagascar, Mauritius and the Seychelles. Created by the Port-Louis

Declaration in 1982, the IOC was institutionalized in Seychelles in 1984 by the General

Agreement for Cooperation, better known as the “Victoria Agreement ”.

• The only regional organization in Africa composed exclusively of islands, it defends

the specificities of its member states on the continental and international scenes.

Benefiting from the active support of a dozen international partners , the IOC

embodies regional solidarity through cooperation projects covering a wide range of

sectors: preservation of ecosystems, sustainable management of natural resources,

maritime security, entrepreneurship, public health , renewable energies or culture.

Page 32: SIMPLYFYING IAS EXAM PREPARATION - INSIGHTSIAS€¦ · This document is the compilation of 100 questions that are part of InsightsIAS famous INSTA REVISION initiative for UPSC civil

INSTA 75 Days REVISION PLAN for Prelims 2020 - InstaTests

www.insightsonindia.com 30 Insights IAS

• In 2016, an observer member status with the IOC was created. IOC observer members

are invited to major organizational meetings and maintain a sustained dialogue on

subjects of common interest.

• IOC has seven observers – India, China, Malta, European Union, International

Organization of La Francophonie (OIF), Japan and United Nations

https://www.commissionoceanindien.org/presentation-coi/

30. Consider the following statements regarding International Cooperation Scheme

1. The Ministry of External Affairs is implementing International Cooperation (IC)

Scheme.

2. It aims to enhance competency of MSMEs, capturing new markets for their

products, exploring new technologies for improving manufacturing capacity.

Which of the statements given above is/are correct?

(a) 1 only

(b) 2 only

(c) Both 1 and 2

(d) Neither 1 nor 2

Solution: B

International Cooperation Scheme:

• The Ministry of Micro, Small and Medium Enterprises (MSME) is implementing

International Cooperation (IC) Scheme.

• Objective: enhance competency of MSMEs, capturing new markets for their

products, exploring new technologies for improving manufacturing capacity, etc.

• Financial assistance is provided under the Scheme on reimbursement basis to the

eligible State /Central Government Organisations, Registered Industry Associations

and Societies/Trusts associated with the promotion and development of MSME sector

to visit/participate in international exhibitions /trade fairs/buyer-seller meet etc.

abroad and also for holding International conferences/seminars/workshops in India

which are in the interest of MSME sector.

Page 33: SIMPLYFYING IAS EXAM PREPARATION - INSIGHTSIAS€¦ · This document is the compilation of 100 questions that are part of InsightsIAS famous INSTA REVISION initiative for UPSC civil

INSTA 75 Days REVISION PLAN for Prelims 2020 - InstaTests

www.insightsonindia.com 31 Insights IAS

31. Consider the following statements regarding Sagar Maitri Mission-2

1. SAGAR MAITRI is a unique initiative of Ministry of Earth Sciences.

2. Its prime objective is data collection from the entire North Indian Ocean, and

establishing long-term collaboration with eight IOR countries in the field of ocean

research and development.

Which of the statements given above is/are correct?

(a) 1 only

(b) 2 only

(c) Both 1 and 2

(d) Neither 1 nor 2

Solution: B

Sagar Maitri Mission-2:

DRDO Research Ship INS Sagardhwani Embarks on Sagar Maitri Mission-2.

SAGAR MAITRI is a unique initiative of DRDO which aligns with the broad objective of Prime

Minister Shri Narendra Modi’s policy declaration “Safety And Growth for All in the Region

(SAGAR)” to promote closer co-operation in socio-economic aspects as well as greater

scientific interaction especially in ocean research among Indian Ocean Rim (IOR) countries.

• Under the aegis of PM’s policy, specific scientific component of DRDO is “MAITRI

(Marine & Allied Interdisciplinary Training and Research Initiative)”.

• SAGAR MAITRI Mission-2 commemorates the Golden Jubilee Celebrations of India’s

lone research ship INS Kistna’s missions as part of the historic International Indian

Ocean Expeditions (IIOE), which took place during 1962-65.

• As part of the mission, INS Sagardhwani will revisit the selected tracks of INS Kistna

and provide NPOL scientists ample opportunities to collaborate and garner a close

working relationship with the oceanographic counterparts of the IOR countries.

• The prime objectives of the SAGAR MAITRI Mission are data collection from the entire

North Indian Ocean, focussing on the the Andaman Sea and adjoining seas and

establishing long-term collaboration with eight IOR countries in the field of ocean

research and development.

• The programme also aims at establishing long term scientific collaboration with these

countries in the field of ‘Ocean Research & Development’ and data collection with a

focus in the Andaman Sea.

Page 34: SIMPLYFYING IAS EXAM PREPARATION - INSIGHTSIAS€¦ · This document is the compilation of 100 questions that are part of InsightsIAS famous INSTA REVISION initiative for UPSC civil

INSTA 75 Days REVISION PLAN for Prelims 2020 - InstaTests

www.insightsonindia.com 32 Insights IAS

32. Consider the following statements regarding North Sea:

1. Germany and France are bordering countries of North Sea

2. Grand Banks, a major fishing ground is located in North Sea

3. River Thames drains into North Sea

Which of the following statements given above is/are correct?

(a) 1 and 2 only

(b) 2 and 3 only

(c) 1 and 3 only

(d) 1, 2 and 3

Solution: C

• The Danube-Oder-Elbe Canal intends to connect the Black, Baltic and North seas for

navigation.

• The North Sea is a marginal sea of the Atlantic Ocean located between Great Britain,

Denmark, Norway, Germany, the Netherlands, Belgium and France.

Page 35: SIMPLYFYING IAS EXAM PREPARATION - INSIGHTSIAS€¦ · This document is the compilation of 100 questions that are part of InsightsIAS famous INSTA REVISION initiative for UPSC civil

INSTA 75 Days REVISION PLAN for Prelims 2020 - InstaTests

www.insightsonindia.com 33 Insights IAS

• The Dogger Bank, a vast moraine, or accumulation of unconsolidated glacial debris,

rises to a mere 15 to 30 m (50 to 100 ft) below the surface. This feature has produced

the finest fishing location of the North Sea.

• River Thames, ancient Tamesis or Tamesa, also called (in Oxford, England) River Isis,

chief river of southern England. Rising in the Cotswold Hills, its basin covers an area

of approximately 5,500 square miles (14,250 square km). Starting as a small trickle in

the Cotswolds the River Thames travels over 210 miles through the heart of some of

England’s most picturesque towns, right into the centre of London and eventually, out

into the North Sea.

• Grand Banks, portion of the North American continental shelf in the Atlantic Ocean,

lying southeast of Newfoundland island, Canada. Noted as an international fishing

ground, the banks extend for 350 miles (560 km) north to south and for 420 miles (675

km) east to west.

Note: Questions have already asked in previous quiz about Baltic Sea and Black Sea.

33. Consider the following statements regarding Spektr-RG

1. It is a joint project between the Russian space agency, Roscosmos, and the

German space agency.

2. A key goal of Spektr-RG will be to investigate the mysterious cosmic components

referred to as “dark matter” and “dark energy”.

Which of the statements given above is/are correct?

(a) 1 only

(b) 2 only

(c) Both 1 and 2

Page 36: SIMPLYFYING IAS EXAM PREPARATION - INSIGHTSIAS€¦ · This document is the compilation of 100 questions that are part of InsightsIAS famous INSTA REVISION initiative for UPSC civil

INSTA 75 Days REVISION PLAN for Prelims 2020 - InstaTests

www.insightsonindia.com 34 Insights IAS

(d) Neither 1 nor 2

Solution: C

Russia Launches Spektr-RG

• Spektr-RG is a Russian–German high-energy astrophysics space observatory

launched recently. It follows on from the Spektr-R satellite telescope launched in

2011.

About Spektr- RG:

• The Spektrum-Röntgen-Gamma mission, also known as Spektr-RG, is a joint project

between the Russian space agency, Roscosmos, and the German space agency, DLR.

• Position: Spektr-RG will be placed in a stable orbit in space called a Lagrange point

(specifically, L2), where the gravitational forces of two large objects — in this case, the

sun and the Earth — balance each other out.

• This location will allow Spektr-RG to perform its observations while using a minimal

amount of fuel.

• Objectives: The spacecraft is expected to detect 100,000 galaxy clusters, 3 million

supermassive black holes, tens of thousands of star-forming galaxies, the presence

of plasma (superheated gas) and many more types of objects.

• The observatory includes two X-ray mirror telescopes, called ART-XC and eROSITA.

• A key goal of Spektr-RG will be to investigate the mysterious cosmic components

referred to as “dark matter” and “dark energy”.

34. Consider the following statements regarding LaQshya initiative

1. It was launched by Ministry of Micro, Small and Medium Enterprises.

2. It aims to improve quality of products developed by MSME’s.

Which of the statements given above is/are correct?

(a) 1 only

(b) 2 only

(c) Both 1 and 2

(d) Neither 1 nor 2

Solution: D

Page 37: SIMPLYFYING IAS EXAM PREPARATION - INSIGHTSIAS€¦ · This document is the compilation of 100 questions that are part of InsightsIAS famous INSTA REVISION initiative for UPSC civil

INSTA 75 Days REVISION PLAN for Prelims 2020 - InstaTests

www.insightsonindia.com 35 Insights IAS

LaQshya

• Ministry of Health and Family Welfare has launched “LaQshya” (Labour room Quality

improvement Initiative) to improve quality of care in labour room and maternity

operation theatres in public health facilities.

About LaQshya:

• It’s a multipronged approach focused at Intrapartum and immediate postpartum

period.

• Aim: To reduce preventable maternal and newborn mortality, morbidity and

stillbirths associated with the care around delivery in Labour room and Maternity

Operation Theatre and ensure respectful maternity care.

Objectives:

• To reduce maternal and newborn mortality & morbidity due to hemorrhage, retained

placenta, preterm, preeclampsia and eclampsia, obstructed labour, puerperal sepsis,

newborn asphyxia, and newborn sepsis, etc.

• To improve Quality of care during the delivery and immediate post-partum care,

stabilization of complications and ensure timely referrals, and enable an effective two-

way follow-up system.

• To enhance satisfaction of beneficiaries visiting the health facilities and provide

Respectful Maternity Care (RMC) to all pregnant women attending the public health

facilities.

Following types of healthcare facilities have been identified for implementation of LaQshya

program:

• Government medical college hospitals.

• District Hospitals & equivalent health facilities.

• Designated FRUs and high case load CHCs with over 100 deliveries/month (60 in hills

and desert areas)

35. Consider the following statements regarding the Kuril Islands:

1. The islands separate the Sea of Japan from the North Pacific Ocean.

2. It is a chain of islands that stretch between the Japanese island of

3. Hokkaido at the southern end and the Russian Kamchatka Peninsula at the

northern end.

4. Iturup island is part of Kuril Islands.

Which of the statements given above is/are correct?

(a) 1 and 2 only

(b) 2 and 3 only

(c) 1 and 3 only

Page 38: SIMPLYFYING IAS EXAM PREPARATION - INSIGHTSIAS€¦ · This document is the compilation of 100 questions that are part of InsightsIAS famous INSTA REVISION initiative for UPSC civil

INSTA 75 Days REVISION PLAN for Prelims 2020 - InstaTests

www.insightsonindia.com 36 Insights IAS

(d) 2 only

Solution: B

Page 39: SIMPLYFYING IAS EXAM PREPARATION - INSIGHTSIAS€¦ · This document is the compilation of 100 questions that are part of InsightsIAS famous INSTA REVISION initiative for UPSC civil

INSTA 75 Days REVISION PLAN for Prelims 2020 - InstaTests

www.insightsonindia.com 37 Insights IAS

• U.S. authorities put out tsunami warnings after an earthquake of magnitude 7.8

struck off Russia’s Kuril Islands, but meteorological officials in Japan issued no alerts,

although they said there might be slight tidal changes

• The Kuril Islands or Kurile Islands is a volcanic archipelago in Russia’s Sakhalin Oblast

that stretches approximately 1,300 km northeast from Hokkaido, Japan to

Kamchatka, Russia, separating the Sea of Okhotsk from the north Pacific Ocean.

• The Kuril Islands is a chain of islands that stretch between the Japanese island of

Hokkaido at the southern end and the Russian Kamchatka Peninsula at the northern

end.

• Four islands – which Russia calls the Southern Kurils and Japan calls the Northern

Territories – are the subject of a 60-year-old dispute between the two nations.

• They are Kunashir (known in Japanese as Kunashiri), Iturup (Etorofu), Shikotan and

the rocky Habomai islets.

• The southernmost islet in the Habomai group lies only a few kilometres off Nemuro

on the Japanese island of Hokkaido.

• Because of the dispute, Russia and Japan have not yet signed a peace treaty to end

World War II.

• Japanese people migrated north to the islands in the 18th and 19th century, including

members of Hokkaido’s minority Ainu community.

• The islands are surrounded by rich fishing grounds and are thought to have offshore

reserves of oil and gas. Rare rhenium deposits have been found on the Kudriavy

volcano on Iturup.

https://www.thehindu.com/news/international/tsunami-warnings-after-magnitude-78-

quake-strikes-near-russias-kuril-islands/article31159658.ece

36. Consider the following statements regarding Kartarpur Sahib pilgrim corridor

1. It will connect Gurdwara Darbar Sahib in India’s Gurdaspur district with Dera Baba

Nanak shrine in Pakistan’s Kartarpur.

2. Pakistan has agreed in principle to allow visa-free, year-long travel to the Sikh

shrine.

3. It was in gurdwara in Kartarpur, that Guru Nanak assembled a Sikh community

and lived for 18 years until his death in 1539.

Which of the statements given above is/are correct?

(a) 1 and 2 only

(b) 2 and 3 only

(c) 1 and 3 only

(d) 1, 2 and 3

Page 40: SIMPLYFYING IAS EXAM PREPARATION - INSIGHTSIAS€¦ · This document is the compilation of 100 questions that are part of InsightsIAS famous INSTA REVISION initiative for UPSC civil

INSTA 75 Days REVISION PLAN for Prelims 2020 - InstaTests

www.insightsonindia.com 38 Insights IAS

Solution: B

Kartarpur Sahib pilgrim corridor

• The second round of talks with Pakistan on the modalities for operationalisation of

the Kartarpur Sahib Corridor was recently held at Wagah, Pakistan.

• Pakistan has agreed in principle to allow visa-free, year-long travel to the Sikh

shrine.

Concerns raised by India:

• India conveyed its concerns to Pakistan on the possible attempts by individuals and

groups to disrupt the Kartarpur Sahib pilgrimage and the possible flooding of the Dera

Baba Nanak due to earth-filled embankment road or a causeway proposed by

Islamabad.

What is the “Kartarpur Corridor” project?

• The corridor – often dubbed as the “Road to Peace” – will connect Gurdwara Darbar

Sahib in Pakistan’s Kartarpur with Dera Baba Nanak shrine in India’s Gurdaspur

district. The construction of the corridor will allow visa-free access to pilgrims from

India. The proposal for the corridor has been on the table since 1988, but tense

relations between the two countries led to the delay.

The shrine:

• The gurdwara in Kartarpur stands on the bank of the Ravi, about 120 km northeast of

Lahore.

• It was here that Guru Nanak assembled a Sikh community and lived for 18 years until

his death in 1539.

• The shrine is visible from the Indian side, as Pakistani authorities generally trim the

elephant grass that would otherwise obstruct the view.

• Indian Sikhs gather in large numbers for darshan from the Indian side, and binoculars

are installed at Gurdwara Dera Baba Nanak.

37. Who among the following signed a Memorandum of Understanding (MoU) to develop

a Broadband Readiness Index (BRI) for Indian States and Union Territories (UT)?

(a) International Telecommunication Union and the Indian Council for Research on

International Economic Relations (ICRIER)

(b) Cellular Operators Association of India and The Department of Telecom (DoT)

(c) NASSCOM and the Department of Telecom (DoT)

(d) The Department of Telecom (DoT) and the Indian Council for Research on

International Economic Relations (ICRIER)

Page 41: SIMPLYFYING IAS EXAM PREPARATION - INSIGHTSIAS€¦ · This document is the compilation of 100 questions that are part of InsightsIAS famous INSTA REVISION initiative for UPSC civil

INSTA 75 Days REVISION PLAN for Prelims 2020 - InstaTests

www.insightsonindia.com 39 Insights IAS

Solution: D

Broadband Readiness Index for states

• DoT and Indian Council for Research on International Economic Relations (ICRIER)

have signed a Memorandum of Understanding (MoU) to develop a Broadband

Readiness Index (BRI) for Indian states and Union Territories (UT).

• The index will include indicators such as percentage of households using computers/

laptops with internet connection, percentage of households with fixed broadband

connection, internet users as a percentage of the population, smartphones density,

percentage of households with at least one digitally literate member, etc.

Objectives:

• Appraise the condition of the underlying digital infrastructure and related factors at

state/UT levels.

• Provide useful insights into strategic choices made by states for investment allocations

in ICT programmes, the statement said.

• Encourage states to cross learn and jointly participate in achieving the overall

objective of digital inclusion and development in India.

Background:

• The National Digital Communication Policy (NDCP) 2018 acknowledged the need for

building a robust digital communications infrastructure leveraging existing assets of

the broadcasting and power sectors including collaborative models involving state,

local bodies and the private sector. The policy recommended that an index for states

and UTs be developed to attract investments and address Right of Way (RoW)

challenges across India.

38. Consider the following statements regarding National Translation Mission

1. The scheme is being implemented through the Sahitya Akedami (National

Academy of Letters)

2. Under the scheme, the books of knowledge texts mostly text books of various

subjects prescribed in Universities and Colleges are being translated in all

Languages of the 8th Schedule of the Constitution of India.

Which of the statements given above is/are correct?

(a) 1 only

(b) 2 only

(c) Both 1 and 2

(d) Neither 1 nor 2

Page 42: SIMPLYFYING IAS EXAM PREPARATION - INSIGHTSIAS€¦ · This document is the compilation of 100 questions that are part of InsightsIAS famous INSTA REVISION initiative for UPSC civil

INSTA 75 Days REVISION PLAN for Prelims 2020 - InstaTests

www.insightsonindia.com 40 Insights IAS

Solution: B

• National Translation Mission is a scheme launched in 2008 which is being

implemented through the Central Institute of Indian Languages (CIIL), Mysore. It aims

to establish translation as an industry in general and to facilitate higher education by

making knowledge texts accessible to students and academics in Indian languages.

• Under the scheme, the books of knowledge texts mostly text books of various subjects

prescribed in Universities and Colleges are being translated in all Languages of the

8th Schedule of the Constitution of India.

39. Consider the following statements regarding Edge computing

1. In this, instead of analyzing the data locally, closer to where it is stored, in real-

time without latency, the data is sent far away to a centralized data centre.

2. The basic difference between edge computing and cloud computing lies in where

the data processing takes place.

3. It allows for quicker data processing and content delivery.

Which of the statements given above is/are correct?

(a) 1 and 2 only

(b) 2 and 3 only

(c) 1 and 3 only

(d) 1, 2 and 3

Solution: B

Edge computing

• According to a research, By 2025, companies will generate and process more than 75%

of their data outside of traditional centralised data centres — that is, at the “edge” of

the cloud.

• As the use of Internet of Things (IoT) devices expands and the need to process IoT

data quickly increases, many IT leaders are considering or beginning to employ edge

computing options.

What is edge computing?

• Edge computing enables data to be analysed, processed, and transferred at the edge

of a network. Meaning , the data is analysed locally, closer to where it is stored, in

real-time without latency, rather than send it far away to a centralised data centre.

Page 43: SIMPLYFYING IAS EXAM PREPARATION - INSIGHTSIAS€¦ · This document is the compilation of 100 questions that are part of InsightsIAS famous INSTA REVISION initiative for UPSC civil

INSTA 75 Days REVISION PLAN for Prelims 2020 - InstaTests

www.insightsonindia.com 41 Insights IAS

• It allows for quicker data processing and content delivery.

How is edge computing different from cloud computing?

• The basic difference between edge computing and cloud computing lies in where the

data processing takes place.

• Internet of Things (IoT) systems perform all of their computations in the cloud using

data centres.

• In Edge computing, massive amounts of data generated by IoT devices are stored and

processed locally. That data doesn’t need to be sent over a network as soon as it

processed; only important data is sent — therefore, an edge computing network

reduces the amount of data that travels over the network.

Benefits:

• Edge computing may offer some protection against a catastrophic attack where a

single incident can compromise large amounts of a company’s data.

• In some ways, it’s more resilient, because instead of one or two or even three data

centers, you have distributed data and compute on the edge, which makes it much

more resilient to malicious and nonmalicious events.

40. Consider the following statements regarding High-Temperature Proton Exchange

Membrane (HTPEM) technology

1. It was developed by the Council of Scientific and Industrial Research (CSIR) in

partnership with Indian industries.

2. It takes hydrogen as the input and produces heat and water as its bi-products.

3. It was built under India’s flagship programme ‘New Millennium Indian Technology

Leadership Initiative (NMITLI)’

Which of the statements given above is/are correct?

(a) 1 and 2 only

(b) 2 and 3 only

(c) 1 and 3 only

(d) 1, 2 and 3

Solution: C

Page 44: SIMPLYFYING IAS EXAM PREPARATION - INSIGHTSIAS€¦ · This document is the compilation of 100 questions that are part of InsightsIAS famous INSTA REVISION initiative for UPSC civil

INSTA 75 Days REVISION PLAN for Prelims 2020 - InstaTests

www.insightsonindia.com 42 Insights IAS

High-Temperature Proton Exchange Membrane (HTPEM) technology

• India’s first indigenously developed high-temperature based Fuel Cell System was

recently introduced. It is a 5.0 kW fuel cell system that generates power in a green

manner.

• The developed fuel cells are based on High-Temperature Proton Exchange

Membrane (HTPEM) technology.

Key facts:

• Developed under the Public-Private Partnership (PPP) model by the Council of

Scientific and Industrial Research (CSIR) in partnership with Indian industries.

• Built under India’s flagship programme named ‘New Millennium Indian Technology

Leadership Initiative (NMITLI)’.

How it works?

• It takes methanol or bio-methane as the input and produces heat and water as its bi-

products, which can be further used. This helps to attain an efficiency that is greater

than 70%, which is difficult to achieve by other energy sources.

Applications:

• Suitable for distributed stationary power applications like; for small offices,

commercial units, data centers etc.; where highly reliable power is essential with

simultaneous requirement for air-conditioning.

• It will also meet the requirement of efficient, clean and reliable backup power

generator for telecom towers, remote locations and strategic applications as well.

• Replace Diesel Generating (DG) sets and help reduce India’s dependence on crude oil.

What is HTPEM technology?

• High Temperature Proton-Exchange-Membrane (HTPEM) is the core of the fuel cells

running above 150 °C. As in classical PEM fuel cells technology, Hydrogen is

electrochemically split to proton and electron on anode. Proton is transported

through membrane to cathode while electricity is yielded in external circuit. At

cathode protons recombine with electron and reacts further with Oxygen to water

and heat.

• The technology can achieve efficiency approaching 90% calculated as combined yield

of electricity and heat.

Significance of HTPEM technology:

• HT-PEM Technology enables simple and cost effective fuel cell systems that can

operate on fuel sources available today, such as natural gas, propane, and methanol.

• When compared to low temperature PEM fuel cells, subsystems for membrane

humidification and carbon monoxide clean up from reformate can be eliminated.

Page 45: SIMPLYFYING IAS EXAM PREPARATION - INSIGHTSIAS€¦ · This document is the compilation of 100 questions that are part of InsightsIAS famous INSTA REVISION initiative for UPSC civil

INSTA 75 Days REVISION PLAN for Prelims 2020 - InstaTests

www.insightsonindia.com 43 Insights IAS

The operation of polymer electrolyte membrane (PEM) fuel cells at temperatures above

150°C offers very significant advantages compared to the conventional PEM fuel cells which

are listed below:

• High CO tolerance

• No need for humidified gases

• High power-to-volume ratio

• Reduced complexity

• Low cost

41. Consider the following statements regarding Nano Technology

1. Nanotechnology allows manipulation of properties of a material at a very small

scale.

2. The physical, chemical and biological properties exhibited by a material changes

at nano scale level in unique and peculiar way.

Which of the statements given above is/are correct?

(a) 1 only

(b) 2 only

(c) Both 1 and 2

(d) Neither 1 nor 2

Solution: C

• The physical, chemical and biological properties exhibited by a material changes at

nano scale level in unique and peculiar way, i.e. it follows the laws of quantum physics

which is very different from the laws of Newtonian physics like gravity, law of motion

etc.,

• As nanotechnology allows manipulation of properties of a material at a very small

scale, scientists and engineers are finding a wide variety of ways to deliberately make

materials at the nanoscale to take advantage of their enhanced properties such as

higher strength, lighter weight, increased control of light spectrum, and greater

chemical reactivity than their larger-scale counterparts. As a result, it has many

applications which is not normally found through conventional material science.

42. Consider the following statements regarding Rare diseases

1. A rare disease shall be a genetic disease that affects a small percentage of the

population.

Page 46: SIMPLYFYING IAS EXAM PREPARATION - INSIGHTSIAS€¦ · This document is the compilation of 100 questions that are part of InsightsIAS famous INSTA REVISION initiative for UPSC civil

INSTA 75 Days REVISION PLAN for Prelims 2020 - InstaTests

www.insightsonindia.com 44 Insights IAS

2. The most common rare diseases recorded in India are Haemophilia, Thalassemia,

sickle-cell anaemia and primary immuno deficiency.

Which of the statements given above is/are correct?

(a) 1 only

(b) 2 only

(c) Both 1 and 2

(d) Neither 1 nor 2

Solution: B

Rare diseases

• The Union Ministry of Health and Family Welfare has published a national policy for

the treatment of 450 ‘rare diseases’.

• The Centre first prepared such a policy in 2017 and appointed a committee in 2018 to

review it.

Overview of the policy:

• Among other measures, the policy intends to kickstart a registry of rare diseases,

which will be maintained by the Indian Council of Medical Research (ICMR).

• According to the policy, rare diseases include genetic diseases, rare cancers,

infectious tropical diseases, and degenerative diseases.

• Under the policy, there are three categories of rare diseases — requiring one-time

curative treatment, diseases that require long-term treatment but where the cost is

low, and those needing long-term treatments with high cost. Some of the diseases in

the first category include osteopetrosis and immune deficiency disorders, among

others.

• As per the policy, the assistance of Rs 15 lakh will be provided to patients suffering

from rare diseases that require a one-time curative treatment under the Rashtriya

Arogya Nidhi scheme. The treatment will be limited to the beneficiaries of Pradhan

Mantri Jan Arogya Yojana.

What is a rare disease?

• A rare disease, also referred to as an orphan disease, is any disease that affects a small

percentage of the population.

• Most rare diseases are genetic, and are present throughout a person’s entire life,

even if symptoms do not immediately appear.

Page 47: SIMPLYFYING IAS EXAM PREPARATION - INSIGHTSIAS€¦ · This document is the compilation of 100 questions that are part of InsightsIAS famous INSTA REVISION initiative for UPSC civil

INSTA 75 Days REVISION PLAN for Prelims 2020 - InstaTests

www.insightsonindia.com 45 Insights IAS

Characteristics:

• Rare diseases are characterised by a wide diversity of symptoms and signs that vary

not only from disease to disease but also from patient to patient suffering from the

same disease. Relatively common symptoms can hide underlying rare diseases,

leading to misdiagnosis.

• The most common rare diseases recorded in India are Haemophilia, Thalassemia,

sickle-cell anaemia and primary immuno deficiency in children, auto-immune

diseases, Lysosomal storage disorders such as Pompe disease, Hirschsprung disease,

Gaucher’s disease, Cystic Fibrosis, Hemangiomas and certain forms of muscular

dystrophies.

Definition:

• While there is no universally accepted definition of rare diseases, countries typically

arrive at their own descriptions, taking into consideration disease prevalence, its

severity and the existence of alternative therapeutic options.

• In the US, for instance, a rare disease is defined as a condition that affects fewer than

200,000 people.

• The same definition is used by the National Organisation for Rare Disorders (NORD).

43. Consider the following statements regarding Biotechnology Industry Research

Assistance Council (BIRAC)

1. It is a not-for-profit Public Sector Enterprise, set up by Department of

Biotechnology (DBT)

2. BIRAC is an industry-academia interface

3. It is an Interface Agency to strengthen and empower the emerging Biotech

enterprise to undertake strategic research and innovation.

Which of the statements given above is/are correct?

(a) 1 and 2 only

(b) 2 and 3 only

(c) 1 and 3 only

(d) 1, 2 and 3

Solution: D

• Biotechnology Industry Research Assistance Council (BIRAC) is a not-for-profit

Section 8, Schedule B, Public Sector Enterprise, set up by Department of

Biotechnology (DBT), Government of India as an Interface Agency to strengthen and

Page 48: SIMPLYFYING IAS EXAM PREPARATION - INSIGHTSIAS€¦ · This document is the compilation of 100 questions that are part of InsightsIAS famous INSTA REVISION initiative for UPSC civil

INSTA 75 Days REVISION PLAN for Prelims 2020 - InstaTests

www.insightsonindia.com 46 Insights IAS

empower the emerging Biotech enterprise to undertake strategic research and

innovation, addressing nationally relevant product development needs.

• BIRAC is a industry-academia interface and implements its mandate through a wide

range of impact initiatives, be it providing access to risk capital through targeted

funding, technology transfer, IP management and handholding schemes that help

bring innovation excellence to the biotech firms and make them globally competitive.

In its Eight years of existence, BIRAC has initiated several schemes, networks and

platforms that help to bridge the existing gaps in the industry-academia Innovation

research and facilitate novel, high quality affordable products development through

cutting edge technologies. BIRAC has initiated partnerships with several national and

global partners to collaborate and deliver the salient features of its mandate.

• BIRAC’s aim is to play a transformative and catalytic role in building a US$ 100 billion

Indian bioeconomy. We believe that the agents of change for building the Indian

bioeconomy would be biotech start-ups & SMEs & hence our focus is on raising their

capabilities

44. Which of the following is/are applications of Graphene?

1. To enhance the strength of the materials.

2. Microelectronics

3. Energy storage

4. Faster DNA Sequencing

Select the correct answer using the code given below

(a) 3 and 4 only

(b) 1, 2 and 3 only

(c) 2, 3 and 4 only

(d) 1, 2, 3 and 4

Solution: D

• Graphene is a one-atom-thick sheet of carbon atoms arranged in a honeycomb-like

pattern. Graphene is considered to be the world’s thinnest, strongest and most

conductive material – to both electricity and heat. All this properties are exciting

researchers and businesses around the world – as graphene has the potential the

revolutionize entire industries – in the fields of electricity, conductivity, energy

generation, batteries, sensors and more.

• Graphene is the world’s strongest material, and so can be used to enhance the

strength of other materials. Dozens of researches have demonstrated that adding

Page 49: SIMPLYFYING IAS EXAM PREPARATION - INSIGHTSIAS€¦ · This document is the compilation of 100 questions that are part of InsightsIAS famous INSTA REVISION initiative for UPSC civil

INSTA 75 Days REVISION PLAN for Prelims 2020 - InstaTests

www.insightsonindia.com 47 Insights IAS

even a trade amount of graphene to plastics, metals or other materials can make these

materials much stronger

• Graphene is the world’s most conductive material to heat. As graphene is also strong

and light, it means that it is a great material to make heat-spreading solutions, such as

heat sinks. This could be useful in both microelectronics (for example to make LED

lighting more efficient and longer lasting) and also in larger applications – for example

thermal foils for mobile devices.

• Because graphene is the world’s thinnest material, it is also the material with the

highest surface-area to volume ratio. This makes graphene a very promising material

to be used in batteries and supercapacitors. Graphene may enable devices that can

store more energy – and charge faster, too. Graphene can also be used to enhance

fuel-cells.

• Graphene has a lot of other promising applications: anti-corrosion coatings and

paints, efficient and precise sensors, faster and efficient electronics, flexible

displays, efficient solar panels, faster DNA sequencing, drug delivery, and more.

45. Consider the following statements regarding Earth System Science Organization (ESSO)

1. It operates as an executive arm of the Ministry of Science and Technology for its

policies and programmes.

2. It was established as a virtual organization, bringing all meteorological and ocean

development activities under one umbrella.

Which of the statements given above is/are correct?

(a) 1 only

(b) 2 only

(c) Both 1 and 2

(d) Neither 1 nor 2

Solution: B

Earth System Science Organization (ESSO)

• The Earth System Science Organization (ESSO) , New Delhi, operates as an executive

arm of the Ministry of Earth Sciences (MoES) for its policies and programmes. The

ESSO provides overall direction for the centres/units and review the implementation

of the programs. The ESSO was established in October, 2007 as a virtual organization,

bringing all meteorological and ocean development activities under one umbrella,

recognizing the importance of strong coupling among various components of the

earth viz. atmosphere, oceans, cryo-sphere and geo-sphere. It has four major

Page 50: SIMPLYFYING IAS EXAM PREPARATION - INSIGHTSIAS€¦ · This document is the compilation of 100 questions that are part of InsightsIAS famous INSTA REVISION initiative for UPSC civil

INSTA 75 Days REVISION PLAN for Prelims 2020 - InstaTests

www.insightsonindia.com 48 Insights IAS

branches of earth sciences, viz. (i) Ocean Science & Technology (ii) Atmospheric and

Climate Science and (iii) Geoscience and Technology and (iv) Polar Science and

Cryosphere. The sole purpose of the endeavour was to address holistically various

aspects relating to earth processes for understanding the variability of earth system.

• The ESSO is primarily aimed to develop and improve capability to forecast, weather,

climate and hazard related phenomena for social, economic and environmental

benefits including addressing aspects relating to climate change science, and climate

services. ESSO is also responsible for development of technology towards the

exploration and exploitation of marine resources in a sustainable way for the socio-

economic benefit of the society by taking into account the global developments in the

field of marine environment. One of the mandates of ESSO is also to promote research

in polar science of both Antarctic and Arctic Regions to understand the various

phenomenon and processes of these regions on global climate and weather, in

particularly on the Indian Ocean.

The overall vision of the ESSO is to excel in knowledge and technology enterprise for the earth

system science realm towards socio-economic benefit of the Indian sub-continent and in the

Indian Ocean region. It has three major components :

1. Provide scientific and technical support for both academic and applied research in

Earth System sciences as a whole comprising the atmosphere, hydrosphere,

cryosphere and the geosphere, with particular reference to the Indian sub-continent

and the surrounding oceans as well as the Polar Regions.

2. Provide the Nation with the best possible services in forecasting the monsoons and

other weather/climate parameters, ocean state including early warnings to natural

disasters like storm surge, earthquakes, tsunamis and other phenomena through well

integrated programs.

3. Support science and technology development for exploration and exploitation of

ocean resources (living and non-living), ensuring their sustainable utilization.

46. Consider the following statements regarding REN21

1. It is an intergovernmental organisation

2. It is based at the United Nations Environment Programme (UNEP)

3. It facilitate knowledge exchange, policy development and joint action towards a

rapid global transition to renewable energy.

Which of the statements given above is/are correct?

(a) 1 and 2 only

(b) 3 only

(c) 2 and 3 only

(d) 1, 2 and 3

Page 51: SIMPLYFYING IAS EXAM PREPARATION - INSIGHTSIAS€¦ · This document is the compilation of 100 questions that are part of InsightsIAS famous INSTA REVISION initiative for UPSC civil

INSTA 75 Days REVISION PLAN for Prelims 2020 - InstaTests

www.insightsonindia.com 49 Insights IAS

Solution: C

• REN21 is the global renewable energy policy multi-stakeholder network that

connects a wide range of key actors. REN21 brings together governments,

nongovernmental organisations, research and academic institutions, international

organisations and industry to learn from one another and build on successes that

advance renewable energy.

• REN21 is an international non-profit association and is based at the United Nations

Environment Programme (UNEP) in Paris, France.

• REN21’s goal is to facilitate knowledge exchange, policy development and joint

action towards a rapid global transition to renewable energy.

• It assists in policy decision making, REN21 provides high quality information, catalyses

discussion and debate and supports the development of thematic networks.

47. Consider the following statements regarding Technology Incubation and Development

of Entrepreneurs (TIDE) Scheme

1. It promotes innovation by nurturing startups in Information Technology,

Communications & Electronics (ICTE) domain.

2. The scheme was initiated by Ministry of Electronics and Information Technology.

Which of the statements given above is/are correct?

(a) 1 only

(b) 2 only

(c) Both 1 and 2

(d) Neither 1 nor 2

Solution: C

Technology Incubation and Development of Entrepreneurs (TIDE) Scheme was put in place

by MeitY in 2008 to promote innovation by nurturing startups in Information Technology,

Communications & Electronics (ICTE) domain. Under the TIDE Scheme, financial assistance is

provided to Institutions of Higher Learning to strengthen their Technology Incubation Centres

for enabling young entrepreneurs to create technology startup companies for commercial

exploitation of technologies developed by them. Enhanced version Technology Incubation

and Development of Entrepreneurs (TIDE 2.0) Scheme is to promote tech entrepreneurship

through financial and technical support to incubators engaged in supporting ICT startups

using emerging technologies such as Internet of Things (IoT), Artificial Intelligence (AI), Block-

chain, Robotics etc.

Page 52: SIMPLYFYING IAS EXAM PREPARATION - INSIGHTSIAS€¦ · This document is the compilation of 100 questions that are part of InsightsIAS famous INSTA REVISION initiative for UPSC civil

INSTA 75 Days REVISION PLAN for Prelims 2020 - InstaTests

www.insightsonindia.com 50 Insights IAS

48. Consider the following statements regarding India’s Nuclear Doctrine

1. Building and maintaining a credible minimum deterrent.

2. Use of nuclear weapons against non-nuclear weapon states.

3. Continued commitment to the goal of a nuclear-weapon-free world, through

global, verifiable and non-discriminatory nuclear disarmament.

Which of the statements given above is/are correct?

(a) 1 and 3 only

(b) 2 only

(c) 3 only

(d) 1 only

Solution: A

• A national nuclear doctrine represents, the collective set of beliefs or principles held

by the nation with regard to the utility of its nuclear weapons. Post 1998 nuclear test,

India came up with a comprehensive nuclear doctrine to clear doubts and

misunderstandings prevailing around world regarding India’s Nuclear weapon policy.

Features of Nuclear Doctrine

• Building and maintaining a credible minimum deterrent.

• A posture of “No First Use” – nuclear weapons will only be used in retaliation against

a nuclear attack on Indian territory or on Indian forces anywhere.

• Nuclear retaliation to a first strike will be massive and designed to inflict unacceptable

damage.

• Nuclear retaliatory attacks can only be authorised by the civilian political leadership

through the Nuclear Command Authority.

• Non-use of nuclear weapons against non-nuclear weapon states.

• However, in the event of a major attack against India, or Indian forces anywhere, by

biological or chemical weapons, India will retain the option of retaliating with

nuclear weapons.

• A continuance of strict controls on export of nuclear and missile related materials

and technologies, participation in the Fissile Material Cutoff Treaty negotiations, and

continued observance of the moratorium on nuclear tests.

• Continued commitment to the goal of a nuclear-weapon-free world, through global,

verifiable and non-discriminatory nuclear disarmament.

Page 53: SIMPLYFYING IAS EXAM PREPARATION - INSIGHTSIAS€¦ · This document is the compilation of 100 questions that are part of InsightsIAS famous INSTA REVISION initiative for UPSC civil

INSTA 75 Days REVISION PLAN for Prelims 2020 - InstaTests

www.insightsonindia.com 51 Insights IAS

49. Consider the following statements regarding Science and Engineering Research Board

(SERB)

1. It is a statutory body set up under Science and Engineering Research Board Act,

2008.

2. It aims at promoting basic research in Science and Engineering and to provide

financial assistance to persons engaged in such research.

3. The Board is chaired by Minister of Science and Technology

Which of the statements given above is/are correct?

(a) 1 and 2 only

(b) 2 and 3 only

(c) 1 and 3 only

(d) 1, 2 and 3

Solution: A

• One of the most notable developments in the S&T sector in the XI Plan has been the

setting up of the Science and Engineering Research Board (SERB) through an Act of

Parliament, viz. the Science and Engineering Research Board Act, 2008. Promoting

basic research in Science and Engineering and to provide financial assistance to

persons engaged in such research, academic institutions, research and development

laboratories, industrial concerns and other agencies for such research and for matters

connected therewith or incidental thereto are the primary and distinctive mandate of

the Board.

• SERB aims to build up best management systems which would match the best global

practices in the area of promotion and funding of basic research.

• The Board is chaired by the Secretary to the Government of India in the Department

of Science and Technology and shall have other senior government officials and

eminent scientists as members.

50. Consider the following statements regarding Neutrino

1. They are one of the most abound particles in the universe.

2. They can even pass through the earth and come out on the other side.

3. Neutrinos carry electrical charge and nearly massless.

Which of the statements given above is/are correct?

(a) 2 only

(b) 1 only

Page 54: SIMPLYFYING IAS EXAM PREPARATION - INSIGHTSIAS€¦ · This document is the compilation of 100 questions that are part of InsightsIAS famous INSTA REVISION initiative for UPSC civil

INSTA 75 Days REVISION PLAN for Prelims 2020 - InstaTests

www.insightsonindia.com 52 Insights IAS

(c) 1, 2 and 3

(d) 1 and 2 only

Solution: D

• Neutrinos are abundantly found in nature. The Sun, the stars and the atmosphere

produce millions of neutrinos every second. Most of these neutrinos pass through

our body and we do not realize it. They can even pass through the earth and come out

on the other side.

• Neutrinos are similar to the more familiar electron, with one crucial difference:

neutrinos do not carry electric charge. Because neutrinos are electrically neutral,

they are not affected by the electromagnetic forces which act on electrons. Neutrino

belongs to Lepton category.

• Neutrinos are affected only by a “weak” sub-atomic force of much shorter range than

electromagnetism, and are therefore able to pass through great distances in matter

without being affected by it.

DAY – 39 (InstaTest-39)

51. The Ulhas River flows in which of the following state?

(a) Odisha

(b) Jharkhand

(c) Maharashtra

(d) Karnataka

Solution: C

• The Ulhas River is a west flowing river in Western India in the state of Maharashtra.

1,050 passengers rescued from stranded train near Mumbai as the Ulhas River near

Badlapur breached its banks and the train halted.

Page 55: SIMPLYFYING IAS EXAM PREPARATION - INSIGHTSIAS€¦ · This document is the compilation of 100 questions that are part of InsightsIAS famous INSTA REVISION initiative for UPSC civil

INSTA 75 Days REVISION PLAN for Prelims 2020 - InstaTests

www.insightsonindia.com 53 Insights IAS

52. Consider the following statements regarding Agni II missile

1. It is a versatile surface-to-surface medium range nuclear-capable missile.

2. The missile has a strike range of 200 km.

3. It is a two-stage missile equipped with advanced high accuracy navigation system

and solid rocket propellant system.

Which of the statements given above is/are correct?

(a) 1 and 2 only

(b) 2 and 3 only

(c) 1 and 3 only

(d) 1, 2 and 3

Solution: C

Agni II missile

• India recently conducted successfully the first night trial of Agni-II, its versatile

surface-to-surface medium range nuclear-capable missile.

• The missile has a strike range of 2000 km.

• Agni-II, an intermediate range ballistic missile (IRBM), has already been inducted into

the armed forces.

Page 56: SIMPLYFYING IAS EXAM PREPARATION - INSIGHTSIAS€¦ · This document is the compilation of 100 questions that are part of InsightsIAS famous INSTA REVISION initiative for UPSC civil

INSTA 75 Days REVISION PLAN for Prelims 2020 - InstaTests

www.insightsonindia.com 54 Insights IAS

• The two-stage missile equipped with advanced high accuracy navigation system, was

guided by a novel state-of-the-art command and control system and propelled by

solid rocket propellant system.

• Agni-II was developed by Advanced Systems Laboratory along with other DRDO

laboratories and integrated by the Bharat Dynamics Limited, Hyderabad.

• Agni-II is part of the Agni series of missiles which includes 700-km range Agni-I, 3,000-

km range Agni-III, Agni-IV and Agni-V.

53. Consider the following statements regarding Vyom Mitra

1. It is female spacefaring humanoid robot.

2. It is developed by the Indian Space Research Organisation to function on-board

the Gaganyaan.

Which of the statements given above is/are correct?

(a) 1 only

(b) 2 only

(c) Both 1 and 2

(d) Neither 1 nor 2

Solution: C

Vyom Mitra

It is ISRO’s first woman astronaut.

• It is a female spacefaring humanoid robot developed by the Indian Space Research

Organisation to function on-board the Gaganyaan, a crewed orbital spacecraft.

• Vyommitra, equipped with a head, two arms and a torso, is built to mimic crew activity

inside the crew module of Gaganyaan.

• Functions: Attaining launch and orbital postures, responding to the environment,

generating warnings, replacing carbon dioxide canisters, operating switches,

monitoring of the crew module, receiving voice commands, responding via speech

(bilingual).

54. Consider the following statements regarding International Whaling Commission

1. All members of International Whaling Commission are signatories to the

International Convention for the Regulation of Whaling (ICRW).

2. Currently it has 89 member countries.

3. India is yet to join as a member of International Whaling Commission

Page 57: SIMPLYFYING IAS EXAM PREPARATION - INSIGHTSIAS€¦ · This document is the compilation of 100 questions that are part of InsightsIAS famous INSTA REVISION initiative for UPSC civil

INSTA 75 Days REVISION PLAN for Prelims 2020 - InstaTests

www.insightsonindia.com 55 Insights IAS

Which of the statements given above is/are correct?

(a) 1 and 2 only

(b) 1 and 3 only

(c) 3 only

(d) 1, 2 and 3

Solution: A

International Whaling Commission

• Recently Japan announced to quit from the International Whaling Commission and

resume commercial whaling.

• The International Whaling Commission is an Inter-governmental Organization whose

purpose is the conservation of whales and the management of whaling.

• It is an international body set up under International Convention for the Regulation

of Whaling (ICRW).

• The IWC currently has 89 member governments from countries all over the world.

• All members are signatories to the International Convention for the Regulation of

Whaling (ICRW).

• ICRW governs the commercial, scientific, and aboriginal subsistence whaling practices

of fifty-nine member nations.

• In 1986, it adopted a moratorium on commercial whaling. This ban still continues.

• This Convention is the legal framework which established the IWC in 1946.

• India joined as a member of International Whaling Commission in 1981.

55. Consider the following statements regarding Project NETRA

1. It is an initiative of ISRO and an early warning system in space to detect debris and

other hazards to Indian satellites.

2. The project will give India its own capability in space situational awareness (SSA).

3. NETRA’s eventual goal is to capture the low earth orbit, where communication

satellites operate.

Which of the statements given above is/are correct?

(a) 1 and 2 only

(b) 2 and 3 only

(c) 1 and 3 only

(d) 1, 2 and 3

Page 58: SIMPLYFYING IAS EXAM PREPARATION - INSIGHTSIAS€¦ · This document is the compilation of 100 questions that are part of InsightsIAS famous INSTA REVISION initiative for UPSC civil

INSTA 75 Days REVISION PLAN for Prelims 2020 - InstaTests

www.insightsonindia.com 56 Insights IAS

Solution: A

Project NETRA

• ISRO has initiated ‘Project NETRA’ – an early warning system in space to detect debris

and other hazards to Indian satellites.

Significance of the project:

• The project will give India its own capability in space situational awareness (SSA) like

the other space powers — which is used to ‘predict’ threats from debris to Indian

satellites.

• NETRA’s eventual goal is to capture the GEO, or geostationary orbit, scene at 36,000

km where communication satellites operate.

• The effort would make India a part of international efforts towards tracking, warning

about and mitigating space debris.

What is Project NETRA (Network for space object Tracking and Analysis)?

• Under the project, the ISRO plans to put up many observational facilities: connected

radars, telescopes; data processing units and a control centre.

• They can, among others, spot, track and catalogue objects as small as 10 cm, up to a

range of 3,400 km and equal to a space orbit of around 2,000 km.

56. Consider the following statements regarding Muktoshri

1. It is arsenic-resistant rice that can be grown in arsenic prone areas.

2. It is developed by International Rice Research Institute.

3. Arsenic is highly toxic in its inorganic form and can cause cancer and skin lesions.

Which of the statements given above is/are correct?

(a) 1 and 2 only

(b) 2 and 3 only

(c) 1 and 3 only

(d) 1, 2 and 3

Solution: C

Muktoshri- arsenic-resistant rice

• West Bengal government’s rice research centre has come up with a new variety of rice

called Muktoshri that can be grown in arsenic prone areas.

Page 59: SIMPLYFYING IAS EXAM PREPARATION - INSIGHTSIAS€¦ · This document is the compilation of 100 questions that are part of InsightsIAS famous INSTA REVISION initiative for UPSC civil

INSTA 75 Days REVISION PLAN for Prelims 2020 - InstaTests

www.insightsonindia.com 57 Insights IAS

• It was developed jointly by the Rice Research Station at Chinsurah, coming under

West Bengal’s Agriculture Department and the National Botanical Research

Institute, Lucknow.

• West Bengal has a high concentration of arsenic in groundwater, with 83 blocks

across seven districts having higher arsenic levels than permissible limits.

Arsenic- Key facts:

• Arsenic is naturally present at high levels in the groundwater of a number of

countries. It is also present in rocks and soils.

• Arsenic is highly toxic in its inorganic form.

Permissible limit:

• World Health Organization’s provisional guideline value for arsenic in drinking water

is 0.01 mg/l (10 μg/l). The permissible limit of arsenic in India in the absence of an

alternative source is 0.05 mg/l (50 μg/l).

Harmful effects:

• Contaminated water used for drinking, food preparation and irrigation of food crops

poses the greatest threat to public health from arsenic.

• Long-term exposure to arsenic from drinking-water and food can cause cancer and

skin lesions.

• It has also been associated with cardiovascular disease and diabetes.

• In utero and early childhood exposure has been linked to negative impacts on

cognitive development and increased deaths in young adults.

57. Consider the following statements regarding Laboratory for the Conservation of

Endangered Species (LaCONES)

1. It is India’s only research facility engaged in conservation and preservation of

wildlife and its resources.

2. It is a part of centre for cellular and molecular biology.

Which of the statements given above is/are correct?

(a) 1 only

(b) 2 only

(c) Both 1 and 2

(d) Neither 1 nor 2

Solution: C

Page 60: SIMPLYFYING IAS EXAM PREPARATION - INSIGHTSIAS€¦ · This document is the compilation of 100 questions that are part of InsightsIAS famous INSTA REVISION initiative for UPSC civil

INSTA 75 Days REVISION PLAN for Prelims 2020 - InstaTests

www.insightsonindia.com 58 Insights IAS

Laboratory for the Conservation of Endangered Species (LaCONES):

• It is India’s only research facility engaged in conservation and preservation of wildlife

and its resources. It was established in 1998 with the help of Central Zoo Authority of

India, CSIR and the government of Andhra Pradesh.

• LaCONES has helped the Mouse Deer Conservation Breeding Centre at Nehru

Zoological Park to increase the population of mouse deer in Telangana forests.

Amrabad forest, for instance, ran out of mouse deer and through the efforts of

LaCONES, the animal is now reintroduced in Amrabad Tiger Reserve.

• Earlier, LaCONES has developed universal DNA based marker for identification of wild

animals from parts and remains. It has a DNA banking of more than 250 species of

mammals, birds and reptiles.

• It is a part of CCMB (centre for cellular and molecular biology).

• It supports both in-situ habitat preservation and Ex-situ conservation through

captive breeding in controlled environment to restock original wild populations.

• It is the only laboratory in India that has developed methods for collection and

cryopreservation of semen and oocytes from wildlife and successfully reproducing

endangered blackbuck, spotted deer and Nicobar pigeons.

58. India’s First e-waste Clinic was set up in

(a) Haryana

(b) Uttar Pradesh

(c) Karnataka

(d) Madhya Pradesh

Solution: D

India’s First e-waste Clinic

• India’s first e-waste clinic for segregating, processing and disposal of waste from

household and commercial units will be set-up in Bhopal, Madhya Pradesh.

• The clinic is being conceived in compliance with the Solid Waste Management Rules,

2016.

• The e-waste clinic would enable segregation, processing and disposal of waste from

both household and commercial units.

Page 61: SIMPLYFYING IAS EXAM PREPARATION - INSIGHTSIAS€¦ · This document is the compilation of 100 questions that are part of InsightsIAS famous INSTA REVISION initiative for UPSC civil

INSTA 75 Days REVISION PLAN for Prelims 2020 - InstaTests

www.insightsonindia.com 59 Insights IAS

59. Consider the following statements regarding MANAV: Human Atlas Initiative

1. It is an initiative of Council of scientific and industrial research.

2. It aims at creating a database network of all tissues in the human body from the

available scientific literature.

Which of the statements given above is/are correct?

(a) 1 only

(b) 2 only

(c) Both 1 and 2

(d) Neither 1 nor 2

Solution: B

MANAV: Human Atlas Initiative

• Department of Biotechnology (DBT) has launched MANAV: Human Atlas Initiative,

towards improving knowledge on human physiology.

What is MANAV: Human Atlas Initiative?

• It is a project funded by DBT.

• Aims at creating a database network of all tissues in the human body from the

available scientific literature.

• It is a project that involves scientific skill development for annotation, science

outreach along with handling big data.

• The programme will involve gaining better biological insights through physiological

and molecular mapping, develop disease models through predictive computing and

have a wholistic analysis and finally drug discovery.

Who can participate in this project?

• The project can be signed up by students who are in their final year graduation and

above. Students from the fields of biochemistry, biotechnology, microbiology, botany,

zoology, bioinformatics, health sciences, systems biologists, pharmacologists and data

sciences can associate with this project.

• Even participants having a science background but not necessarily involved in active

scientific research can be part of this network.

What are the applications of information generated through MANAV?

• The aim of the project remains to understand and capture the human physiology in

two stages – in a normal stage and while in a disease stage. Such a database on

individual tissues, once ready, can come handy in tracing the causes of a disease,

Page 62: SIMPLYFYING IAS EXAM PREPARATION - INSIGHTSIAS€¦ · This document is the compilation of 100 questions that are part of InsightsIAS famous INSTA REVISION initiative for UPSC civil

INSTA 75 Days REVISION PLAN for Prelims 2020 - InstaTests

www.insightsonindia.com 60 Insights IAS

understanding specific pathways and ultimately decode the body’s disease stage

linked to tissues and cells. The teams will also study any potent elements or molecules

that have never been used in the form of drugs, to target the specific cells or tissues.

60. Consider the following statements regarding Project Elephant

1. It is a centrally sponsored scheme

2. It provides financial and technical support to major elephant bearing states in the

country for protection of elephants, their habitats and corridors.

3. It addresses the issues of human-elephant conflict and welfare of domesticated

elephants.

Which of the statements given above is/are correct?

(a) 2 and 3 only

(b) 1 and 3 only

(c) 1 and 2 only

(d) 1, 2 and 3

Solution: D

Project Elephant:

• It is a centrally sponsored scheme, launched in 1992 by Ministry of Environment and

Forests.

• It provides financial and technical support to major elephant bearing states in the

country for protection of elephants, their habitats and corridors.

• It also seeks to address the issues of human-elephant conflict and welfare of

domesticated elephants.

Objectives:

• To protect elephants, their habitat & corridors

• To address issues of man-animal conflict

• Welfare of captive elephants

Major activities:

• Ecological restoration of existing natural habitats and migratory routes of elephants;

• Development of scientific and planned management for conservation of elephant

habitats and viable population of Wild Asiatic elephants in India;

Page 63: SIMPLYFYING IAS EXAM PREPARATION - INSIGHTSIAS€¦ · This document is the compilation of 100 questions that are part of InsightsIAS famous INSTA REVISION initiative for UPSC civil

INSTA 75 Days REVISION PLAN for Prelims 2020 - InstaTests

www.insightsonindia.com 61 Insights IAS

• Promotion of measures for mitigation of man elephant conflict in crucial habitats and

moderating pressures of human and domestic stock activities in crucial elephant

habitats;

• Strengthening of measures for protection of Wild elephants form poachers and

unnatural causes of death;

• Eco-development and Veterinary care.

61. StrandHogg often seen in news is a

(a) Malware

(b) Robot for narcotics detection

(c) New particle discovered in Large hadron collider.

(d) Fossil of the oldest known mammal

Solution: A

StrandHogg

• It is a Malware which allows real-time malware applications to pose as genuine

applications and access user data of all kinds.

• Threats: It can listen to the conversations, access photo album, read/send messages,

make calls, record conversations, get login credentials to various accounts, access

private images, files, contact details, call logs and location information without being

apparent to the affected users.

• Why in News? The Union Home Ministry has alerted States, warning them about the

vulnerability of the Android operating system to a bug called ‘StrandHogg’.

62. Consider the following pairs of joint military exercise and the countries performed

1. Ekuverin : India and Thailand 2. Dharma Guardian : India and Japan 3. Shakti : India and France 4. Dustlik : India and Kazakhstan

Which of the pairs given above is/are correctly matched?

(a) 1 and 2 only

(b) 2 and 3 only

(c) 3 and 4 only

(d) 1 and 4 only

Page 64: SIMPLYFYING IAS EXAM PREPARATION - INSIGHTSIAS€¦ · This document is the compilation of 100 questions that are part of InsightsIAS famous INSTA REVISION initiative for UPSC civil

INSTA 75 Days REVISION PLAN for Prelims 2020 - InstaTests

www.insightsonindia.com 62 Insights IAS

Solution: B

Joint Military Exercises involving India:

• Ekuverin – It is a joint military exercise between Indian and Maldives.

• Nomadic Elephant – It is Indo – Mongolian joint military training.

• Maitree – 2019 – It is the joint military exercise between India and Thailand.

• Dharma Guardian – 2019 – It is a joint military exercise between India and Japan.

• Shakti-2019 – Exercise Shakti is a series of joint military exercise between India and

France.

• Dustlik-2019 – It is the first ever India-Uzbekistan joint military exercise.

• Kazind – It is an annual military exercise between India and Kazakhstan.

63. Which of the following countries is not part of Golden Crescent, one of the Asia’s

principal areas of illicit opium production?

(a) India

(b) Iran

(c) Pakistan

(d) Afghanistan

Solution: A

Page 65: SIMPLYFYING IAS EXAM PREPARATION - INSIGHTSIAS€¦ · This document is the compilation of 100 questions that are part of InsightsIAS famous INSTA REVISION initiative for UPSC civil

INSTA 75 Days REVISION PLAN for Prelims 2020 - InstaTests

www.insightsonindia.com 63 Insights IAS

• The Golden Crescent is the name given to one of Asia’s two principal areas of illicit

opium production (with the other being the Golden Triangle). The Golden Crescent is

located in Southwest Asia and consists of three contiguous countries of Pakistan,

Afghanistan and Iran from East to West.

• Afghanistan, the only land locked country in this region, has the least area of 6,47,497

sq. km. Iran having the largest coastline of 2440 km in this region is marked by) Persian

Gulf and Gulf of Oman in the South. Pakistan shares the maximum land boundaries

with a total boundary area of 6,774 km.

• Iran is not an opium producing country in this region but is more that of a trans-

shipment country.

64. Consider the following statements regarding Paramarsh scheme

1. The scheme is for Mentoring National Accreditation and Assessment Council

(NAAC) Accreditation Aspirant Institutions.

2. The Scheme will be operationalized through a “Hub & Spoke” model.

Which of the statements given above is/are correct?

(a) 1 only

(b) 2 only

(c) Both 1 and 2

(d) Neither 1 nor 2

Page 66: SIMPLYFYING IAS EXAM PREPARATION - INSIGHTSIAS€¦ · This document is the compilation of 100 questions that are part of InsightsIAS famous INSTA REVISION initiative for UPSC civil

INSTA 75 Days REVISION PLAN for Prelims 2020 - InstaTests

www.insightsonindia.com 64 Insights IAS

Solution: C

Paramarsh scheme

• The Union Ministry of Human Resource Development has launched ‘Paramarsh’ – a

University Grants Commission (UGC) scheme.

About Paramarsh:

• The scheme is for Mentoring National Accreditation and Assessment Council (NAAC)

Accreditation Aspirant Institutions to promote Quality Assurance in Higher Education.

• The scheme will be a paradigm shift in the concept of mentoring of institution by

another well performing institution to upgrade their academic performance and

enable them to get accredited by focusing in the area of curricular aspects, teaching-

learning & evaluation, research, innovation, institutional values & practices etc.

Implementation:

• The Scheme will be operationalized through a “Hub & Spoke” model wherein the

Mentor Institution, called the “Hub” is centralized and will have the responsibility of

guiding the Mentee institution through the secondary branches the “Spoke” through

the services provided to the mentee for self-improvement.

Significance and impact of the scheme:

• The scheme is expected to have a major impact in addressing a national challenge of

improving the quality of Higher Education in India.

• This allows a centralized control over operational efficiency, resource utilization to

attain overall development of the mentee institution.

• The scheme will lead to enhancement of overall quality of the Mentee Institutions and

enhance its profile as a result of improved quality of research, teaching and learning

methodologies.

• Mentee Institution will also have increased exposure and speedier adaptation to best

practices.

• The scheme will also facilitate sharing of knowledge, information and opportunities

for research collaboration and faculty development in Mentee Institutions.

• Mentor-Mentee relationship will not only benefit both the institutions but also

provide quality education to the 3.6 crore students who are enrolling to Indian Higher

Education system at present.

65. Consider the following statements regarding Seva Bhoj Yojana

1. It is a scheme to reimburse central share of CGST and IGST on food, prasad, langar

or bhandara offered by religious and charitable institutions.

2. It was launched by Ministry of Finance.

3. All religious institutions are covered under this scheme.

Page 67: SIMPLYFYING IAS EXAM PREPARATION - INSIGHTSIAS€¦ · This document is the compilation of 100 questions that are part of InsightsIAS famous INSTA REVISION initiative for UPSC civil

INSTA 75 Days REVISION PLAN for Prelims 2020 - InstaTests

www.insightsonindia.com 65 Insights IAS

Which of the statements given above is/are correct?

(a) 1 only

(b) 2 and 3 only

(c) 1 and 2 only

(d) 1, 2 and 3

Solution: A

About Seva Bhoj Yojana:

• Union Ministry of Culture has launched- ‘Seva Bhoj Yojna’– a scheme to reimburse

central share of CGST and IGST on food, prasad, langar or bhandara offered by

religious and charitable institutions.

• The scheme seeks to reimburse the central government’s share of Central Goods and

Services Tax (CGST) and Integrated Goods and Service Tax (IGST) on purchase of raw

items such as ghee, edible oil, atta, maida, rava, flour, rice pulses, sugar and jaggery,

which go into preparation of food/prasad/langar/bhandara offered free of cost by

religious institutions.

• The main objective of the scheme is to lessen the financial burden of such charitable

religious institutions, which provide free of cost without any discrimination to the

general public and devotees.

Eligibility:

• The charitable religious institutions including temples, gurudwara, mosque, church,

dharmik ashram, dargah, monasteries, which fulfill the following criteria are eligible

for the grant:

• The institutions that have been in existence for at least five years before applying for

financial assistance/grant.

• The institutions that serve free food to at least 5000 people in a month.

• The institutions covered under Section 10(23BBA) of the Income Tax Act or those

registered as Society under Societies Registration Act (XXI of 1860) or as a Public Trust

under any law for the time being in force of statuary religious bodies constituted under

any Act or institutions registered under Section 12AA of Income Tax Act.

66. Low enriched uranium bank, sometime seen in the news, is established in which of the

following countries?

(a) Russia

(b) United States of America

Page 68: SIMPLYFYING IAS EXAM PREPARATION - INSIGHTSIAS€¦ · This document is the compilation of 100 questions that are part of InsightsIAS famous INSTA REVISION initiative for UPSC civil

INSTA 75 Days REVISION PLAN for Prelims 2020 - InstaTests

www.insightsonindia.com 66 Insights IAS

(c) France

(d) None of the above

Solution: D

• The IAEA LEU Bank will be located at the Ulba Metallurgical Plant in Oskemen,

Kazakhstan and will be under the responsibility of the appropriate authorities for

safety, security and safeguards in Kazakhstan.

• The IAEA Low Enriched Uranium (LEU) Bank is an assurance of supply mechanism of

last resort, and will be a physical reserve of LEU available for eligible IAEA Member

States.

• The IAEA LEU Bank will be a reserve of LEU owned and controlled by the IAEA, and a

mechanism of last resort for Member States in case the supply of LEU to a nuclear

power plant is disrupted due to exceptional circumstances and the Member State is

unable to secure LEU from the commercial market or by any other means.

https://www.iaea.org/topics/iaea-low-enriched-uranium-bank

67. Consider the following statements regarding Indian Science Congress

1. Indian Science Congress is organized by the Indian Science Congress Association

once in two years in the first week of January.

2. The Indian Science Congress Association was started in the year 1914.

Which of the statements given above is/are correct?

(a) 1 only

(b) 2 only

(c) Both 1 and 2

(d) Neither 1 nor 2

Solution: B

Indian Science Congress

• 107th Indian Science Congress is being held at the University of Agricultural Sciences

(UAS) in Bengaluru, Karnataka.

• Theme: “Science & Technology: Rural Development”.

Page 69: SIMPLYFYING IAS EXAM PREPARATION - INSIGHTSIAS€¦ · This document is the compilation of 100 questions that are part of InsightsIAS famous INSTA REVISION initiative for UPSC civil

INSTA 75 Days REVISION PLAN for Prelims 2020 - InstaTests

www.insightsonindia.com 67 Insights IAS

Background:

• Indian Science Congress is organised by the Indian Science Congress Association every

year in the first week of January.

About Indian Science Congress Association:

• The Indian Science Congress Association was started in the year 1914 in Kolkata and

has a membership of more than 30,000 scientists.

• Origin: It owes its origin to the foresight and initiative of two British chemists, namely,

Professor J. L. Simonsen and Professor P. S. MacMahon. It occurred to them that

scientific research in India might be stimulated if an annual meeting of research

workers somewhat on the lines of the British Association for the Advancement of

Science could be arranged.

Objectives:

• To advance and promote the cause of science in India.

• To hold an annual congress at a suitable place in India.

• To publish such proceedings, journals, transactions and other publications as may be

considered desirable.

• To secure and manage funds and endowments for the promotion of Science including

the rights of disposing of or selling all or any portion of the properties of the

Association.

• To do and perform any or all other acts, matters and things as are conductive to, or

incidental to, or necessary for, the above objects.

68. Consider the following statements regarding Geographical indicators

1. Its time period is 10 years and cannot be renewed again.

2. It is given to both man-made and natural products.

3. GI is covered as element of intellectual property rights (IPRs) under Paris

Convention for Protection of Industrial Property.

Which of the statements given above is/are correct?

(a) 1 and 2 only

(b) 2 and 3 only

(c) 1 and 3 only

(d) 1, 2 and 3

Solution: B

Page 70: SIMPLYFYING IAS EXAM PREPARATION - INSIGHTSIAS€¦ · This document is the compilation of 100 questions that are part of InsightsIAS famous INSTA REVISION initiative for UPSC civil

INSTA 75 Days REVISION PLAN for Prelims 2020 - InstaTests

www.insightsonindia.com 68 Insights IAS

Geographical indicators (GI)

• A name or sign used on goods that have a specific geographical origin and posses

qualities or a reputation due to that place of origin.

• Its purpose is to create unique identification to customers and thereby creating more

demand for products.

• It is given to both man-made and natural products.

• However it is a community right rather than individual or company.

• It is protected under Geographical indications of goods (registration and protection)

act, 1999.

• It is managed by Cell for IPR promotion and Management (CIPAM) under the

Department of Industrial Policy and Promotion (DIPP), Ministry of Commerce and

Industry.

• At the International level, GI is governed by World Trade Organisation‘s (WTO‘s)

Agreement on Trade- Related Aspects of Intellectual Property Rights (TRIPS).

• TIME PERIOD – 10 years + renewed for any time

• The first product in India to be accorded with GI tag was Darjeeling tea in the year

2004-05.

• Once the GI protection is granted, no other producer can misuse the name to market

similar products. It also provides comfort to customers about the authenticity of that

product.

• Provisions in this regard: GI is covered as element of intellectual property rights (IPRs)

under Paris Convention for Protection of Industrial Property.

• At international level, GI is governed by WTO’s Agreement on Trade-Related Aspects

of Intellectual Property Rights (TRIPS). In India, Geographical Indications of Goods

(Registration and Protection Act), 1999 governs it.

69. Which of the following is/are the applications of Big Data?

1. Trade Surveillance

2. Supply chain management

3. Drug discovery

4. Disease pattern analysis

Select the correct answer using the code given below

(a) 1 and 2 only

(b) 2 only

(c) 3 and 4 only

(d) 1, 2, 3 and 4

Solution: D

Page 71: SIMPLYFYING IAS EXAM PREPARATION - INSIGHTSIAS€¦ · This document is the compilation of 100 questions that are part of InsightsIAS famous INSTA REVISION initiative for UPSC civil

INSTA 75 Days REVISION PLAN for Prelims 2020 - InstaTests

www.insightsonindia.com 69 Insights IAS

• Data is information processed or stored by a computer. This information may be in

the form of text documents, images, audio clips, software programs, or other types of

data.

• Big Data is also a data but with a huge size, such a data is so large and complex that

none of the traditional data management tools are able to store it or process it

efficiently.

Applications of Big Data

70. Consider the following statements regarding e-Cigarettes

1. Electronic cigarettes are devices that do not burn or use tobacco leaves but

instead vaporise a solution, which a user then inhales.

2. The main constituents of the solution is nicotine.

3. They are approved as NRTs (nicotine-replacement therapies) under the Drugs and

Cosmetics Act and rules.

Which of the statements given above is/are correct?

(a) 1 and 2 only

(b) 2 and 3 only

(c) 1 and 3 only

(d) 1, 2 and 3

Solution: A

Page 72: SIMPLYFYING IAS EXAM PREPARATION - INSIGHTSIAS€¦ · This document is the compilation of 100 questions that are part of InsightsIAS famous INSTA REVISION initiative for UPSC civil

INSTA 75 Days REVISION PLAN for Prelims 2020 - InstaTests

www.insightsonindia.com 70 Insights IAS

e-Cigarettes

• The Central government has directed all states and Union Territories to not allow the

manufacture, sale and advertisement of e-cigarettes and other Electronic Nicotine

Delivery Systems (ENDS).

• Electronic cigarettes or e-cigarettes, are devices that do not burn or use tobacco

leaves but instead vaporise a solution, which a user then inhales.

• The main constituents of the solution, in addition to nicotine, are propylene glycol

(with or without glycerol and flavouring agents).

• ENDS are devices that heat a solution to create an aerosol, which also frequently

contains flavours, usually dissolved into propylene glycolor and glycerin.

• A number of metals, including lead, chromium and nickel, and chemicals like

formaldehyde have been found in aerosols of some ENDS.

• ENDS are not approved as NRTs (nicotine-replacement therapies) under the Drugs

and Cosmetics Act and rules made thereunder.

• According to Global Tobacco Epidemic 2017 report by the WHO, the governemnts of

30 countries like Mauritius, Australia, Singapore etc have already banned ENDS.

71. Consider the following statements regarding Bitcoin

1. It is a form of digital currency or virtual currency created and held electronically.

2. It is generally regulated by central bank.

3. It can be traded on an open market and its exchange rate fluctuates much like a

stock market

Which of the statements given above is/are correct?

(a) 3 only

(b) 1 and 3 only

(c) 2 only

(d) 1, 2 and 3

Solution: B

Bitcoin is a form of digital currency or virtual currency created and held electronically.

• It is not regulated by any central bank or government. They aren’t printed, like dollars

or euros – they’re produced by people, and increasingly businesses, running

computers all around the world, using software that solves mathematical problems.

• It is also called a “cryptocurrency” since it is decentralized and uses cryptography to

prevent double-spending, a significant challenge inherent to digital currencies.

Page 73: SIMPLYFYING IAS EXAM PREPARATION - INSIGHTSIAS€¦ · This document is the compilation of 100 questions that are part of InsightsIAS famous INSTA REVISION initiative for UPSC civil

INSTA 75 Days REVISION PLAN for Prelims 2020 - InstaTests

www.insightsonindia.com 71 Insights IAS

• Bitcoin is a distributed peer-to-peer digital currency that functions without the inter-

mediation of any central authority.

• It can also be traded on an open market and its exchange rate fluctuates much like

a stock market i.e. based on the demand.

72. Consider the following pairs of the state butterfly and their respective states

1. Maharashtra : Blue Mormon 2. Kerala : Budha Mayoori 3. Karnataka : Southern bird wings 4. Uttarakhand : Common peacock

Which of the pairs given above is/are correctly matched?

(a) 1, 2 and 3 only

(b) 2, 3 and 4 only

(c) 1, 3 and 4 only

(d) 1, 2, 3 and 4

Solution: D

Tamil yeoman:

• Tamil yeoman (Cirrochroa thais) butterfly species endemic to Western Ghats has

been declared the state butterfly of Tamil Nadu.

Key facts:

• Uniformly orange in colour with a dark brown outer ring, Tamil Yeoman is among the

32 butterfly species found in the Western Ghats.

• This butterfly species moves in groups in large numbers, but only in a few places. Also

known as Tamil Maravan, which means warrior, these butterflies are found mainly in

the hilly areas.

• For the first time Tamil Nadu has declared its state butterfly and only fifth in the

country to do so. Maharashtra was the first to declare Blue Mormon as its state

butterfly, followed by Uttarakhand (Common peacock), Karnataka (Southern bird

wings) and Kerala (Malabar banded peacock or Budha Mayoori).

73. Consider the following statements regarding World Nuclear Association

1. It promotes nuclear power and supports the companies that comprise the global

nuclear industry.

Page 74: SIMPLYFYING IAS EXAM PREPARATION - INSIGHTSIAS€¦ · This document is the compilation of 100 questions that are part of InsightsIAS famous INSTA REVISION initiative for UPSC civil

INSTA 75 Days REVISION PLAN for Prelims 2020 - InstaTests

www.insightsonindia.com 72 Insights IAS

2. Its members come from all parts of the nuclear fuel cycle from uranium mining to

the disposition of used nuclear fuel.

Which of the statements given above is/are correct?

(a) 1 only

(b) 2 only

(c) Both 1 and 2

(d) Neither 1 nor 2

Solution: C

The World Nuclear Association (WNA) is the international organization that promotes

nuclear power and supports the companies that comprise the global nuclear industry.

• Its members come from all parts of the nuclear fuel cycle, including uranium mining,

uranium conversion, uranium enrichment, nuclear fuel fabrication, plant

manufacture, transport, and the disposition of used nuclear fuel as well as electricity

generation itself.

• Together, WNA members are responsible for 70% of the world’s nuclear power as

well as the vast majority of world uranium, conversion and enrichment production.

• The WNA says it aims to fulfill a dual role for its members: Facilitating their interaction

on technical, commercial and policy matters and promoting wider public

understanding of nuclear technology. It has a secretariat of around 35 staff. The WNA

was founded in 2001 on the basis of the Uranium Institute, itself founded in 1975.

74. Consider the following statements regarding Chandipura virus

1. It is named after the Gujrat village where the virus was first discovered.

2. The likely vector (carrier) of the virus is the female phlebotomine sandfly.

3. The virus predominantly infects children between the ages of 2-16.

Which of the statements given above is/are correct?

(a) 1 and 2 only

(b) 2 and 3 only

(c) 1 and 3 only

(d) 1, 2 and 3

Page 75: SIMPLYFYING IAS EXAM PREPARATION - INSIGHTSIAS€¦ · This document is the compilation of 100 questions that are part of InsightsIAS famous INSTA REVISION initiative for UPSC civil

INSTA 75 Days REVISION PLAN for Prelims 2020 - InstaTests

www.insightsonindia.com 73 Insights IAS

Chandipura virus:

• Chandipura virus detected in Gujarat.

• Named after the Maharashtra village where the virus was first discovered, the likely

vector (carrier) of the virus is the female phlebotomine sandfly. It has been detected

in sand flies in Senegal and Nigeria, apart from India. The virus is known to cause

inflammation of the brain, and progresses rapidly from an influenza-like illness to

coma and death.

• Chandipura virus (CHPV) belongs to the Rhabdoviridae family in the order

Mononegavirales of the genus Vesiculovirus. Interestingly, its continuing mutating

trend has enhanced its lethality to cause human infections, unlike its genetic cousin,

the vesicular stomatitis virus (VSV).

Symptoms:

• Sudden high fever accompanied by headaches and altered consciousness.

• Convulsions.

• Vomiting and nausea.

• Unconsciousness

Key facts:

• The virus predominantly infects children between the ages of 2-16, spreading through

the bite of a sandfly, and in some cases, even the mosquito during the monsoon and

pre-monsoon season.

• It is distantly related to the virus that causes rabies and is known to have a case fatality

between 55-75 per cent.

75. Consider the following statements regarding Three Stage Nuclear programme in India

1. Pressurized heavy water reactors are used in second stage.

2. Thorium is used as fuel/fissile material in third stage

Which of the statements given above is/are correct?

(a) 1 only

(b) 2 only

(c) Both 1 and 2

(d) Neither 1 nor 2

Solution: B

Page 76: SIMPLYFYING IAS EXAM PREPARATION - INSIGHTSIAS€¦ · This document is the compilation of 100 questions that are part of InsightsIAS famous INSTA REVISION initiative for UPSC civil

INSTA 75 Days REVISION PLAN for Prelims 2020 - InstaTests

www.insightsonindia.com 74 Insights IAS

Three Phases of Indian nuclear power programme

• Homi Bhabha envisioned India’s nuclear power programme in three stages to suit

country’s low uranium resource profile.

Phase I – Pressurized Heavy Water Reactor:

• In the first stage of the programme natural uranium fuelled Pressurized Heavy water

reactors produces electricity while generating plutonium-239 as by-product.

Phase II – Fast Breeder Reactor:

• In the second stage, fast breeder reactors (FBRs) [moderators not required] would use

plutonium239, recovered by reprocessing spent fuel from the first stage, and natural

uranium.

• In FBRs, plutonium-239 undergoes fission to produce energy, while the uranium-238

present in the fuel transmutes to additional plutonium-239.

Phase III – Thorium based Reactors:

• A Stage III reactor or an advanced nuclear power system involves a self-sustaining

series of thorium-232-uranium-233 fuelled reactors.

• This would be a thermal breeder reactor, which in principle can be refueled – after its

initial fuel charge – using only naturally occurring thorium.

As there is a long delay before direct thorium utilization in the three-stage programme, the

country is now looking at reactor designs that allow more direct use of thorium in parallel

with the sequential three-stage programme

Advanced Heavy Water Reactor (AHWR) is such an option under consideration.

DAY – 40 (InstaTest-40)

76. Consider the following statements regarding Basavanna, 12th century philosopher

1. He spread social awareness through his poetry, popularly known as Vachanaas

2. He rejected gender discrimination, superstitions and rituals.

3. He believed in idol worship of the god.

Which of the statements given above is/are correct?

(a) 1 and 3 only

(b) 2 only

(c) 1 and 2 only

(d) 1 only

Page 77: SIMPLYFYING IAS EXAM PREPARATION - INSIGHTSIAS€¦ · This document is the compilation of 100 questions that are part of InsightsIAS famous INSTA REVISION initiative for UPSC civil

INSTA 75 Days REVISION PLAN for Prelims 2020 - InstaTests

www.insightsonindia.com 75 Insights IAS

Solution: C

• Basavanna was an Indian 12th century Statesman, philosopher, poet, Lingayat saint in

the Shiva-focussed Bhakti movement, and social reformer during the reign of the

Kalyani Chalukya/Kalachuri dynasty. Basavanna was active during the rule of both

dynasties but reached his peak of influence during the rule of King Bijjala II in

Karnataka, India.

• Basavanna spread social awareness through his poetry, popularly known as

Vachanaas.

• Basavanna rejected gender or social discrimination, superstitions and rituals but

introduced Ishtalinga necklace, with an image of the Shiva Liṅga, to every person

regardless of his or her birth, to be a constant reminder of one’s bhakti (devotion) to

Shiva.

• As the chief minister of his kingdom, he introduced new public institutions such as the

Anubhava Mantapa (or, the “hall of spiritual experience”), which welcomed men and

women from all socio-economic backgrounds to discuss spiritual and mundane

questions of life, in open

77. Which of the following is/are the features of Charter Act of 1793

1. It gave the Governor-General more powers and control over the governments of

the subordinate Presidencies of Bombay and Madras.

2. It laid down that the members of the Board of Control and their staff were to be

paid out of the British revenues.

3. It extended the trade monopoly of the Company in India for another period of

twenty years.

Select the correct answer using the code given below:

(a) 1 and 2 only

(b) 2 and 3 only

(c) 1 and 3 only

(d) 1, 2 and 3

Solution: C

Charter Act of 1793

• It extended the overriding power given to Lord Cornwallis over his council, to all future

Governor-Generals and Governors of Presidencies.

Page 78: SIMPLYFYING IAS EXAM PREPARATION - INSIGHTSIAS€¦ · This document is the compilation of 100 questions that are part of InsightsIAS famous INSTA REVISION initiative for UPSC civil

INSTA 75 Days REVISION PLAN for Prelims 2020 - InstaTests

www.insightsonindia.com 76 Insights IAS

• It gave the Governor-General more powers and control over the governments of the

subordinate Presidencies of Bombay and Madras.

• It extended the trade monopoly of the Company in India for another period of twenty

years.

• It provided that the Commander-in-Chief was not to be a member of the Governor-

General’s council, unless he was so appointed.

• It laid down that the members of the Board of Control and their staff were, henceforth,

to be paid out of the Indian revenues.

78. The provisions relating to the emoluments, allowances, privileges under Second

Schedule are covered for

1. The Judges of the Supreme Court

2. The Comptroller and Auditor-General of India

3. The members of Parliament

4. The state ministers

Select the correct answer using the code given below:

(a) 1 and 2 only

(b) 1, 2 and 3 only

(c) 3 and 4 only

(d) 1, 2, 3 and 4

Solution: A

Second Schedule

Provisions relating to the emoluments, allowances, privileges and so on of:

• The President of India

• The Governors of States

• The Speaker and the Deputy Speaker of the Lok Sabha

• The Chairman and the Deputy Chairman of the Rajya Sabha

• The Speaker and the Deputy Speaker of the Legislative Assembly in the states

• The Chairman and the Deputy Chairman of the Legislative Council in the states

• The Judges of the Supreme Court

• The Judges of the High Courts

• The Comptroller and Auditor-General of India

Page 79: SIMPLYFYING IAS EXAM PREPARATION - INSIGHTSIAS€¦ · This document is the compilation of 100 questions that are part of InsightsIAS famous INSTA REVISION initiative for UPSC civil

INSTA 75 Days REVISION PLAN for Prelims 2020 - InstaTests

www.insightsonindia.com 77 Insights IAS

79. Consider the following statements regarding sovereign gold bonds (SBGs)

1. They are government securities denominated in grams of gold.

2. Bonds are issued by Non-Banking Financial Institutions and Private Banks in India

3. SGB is free from issues like making charges and purity in the case of gold in jewelry

form.

Which of the statements given above is/are correct?

(a) 1 and 3 only

(b) 2 only

(c) 1 and 2 only

(d) 1, 2 and 3

Solution: A

• SGBs are government securities denominated in grams of gold. They are substitutes

for holding physical gold.

• Investors have to pay the issue price in cash and the bonds will be redeemed in cash

on maturity. The Bond is issued by Reserve Bank on behalf of Government of India.

Benefits of Sovereign Gold Bonds

• Investors are assured of the market value of gold at the time of maturity and periodical

interest.

• SGB is free from issues like making charges and purity in the case of gold in jewellery

form.

• The bonds are held in the books of the RBI or in demat form eliminating risk of loss of

scrip etc.

80. Consider the following statements regarding meaning of term ‘Republic’ used in the

Indian constitution

1. Vesting of political sovereignty in the people and not in a single individual

2. The absence of any privileged class

Which of the statements given above is/are correct?

(a) 1 only

(b) 2 only

(c) Both 1 and 2

(d) Neither 1 nor 2

Page 80: SIMPLYFYING IAS EXAM PREPARATION - INSIGHTSIAS€¦ · This document is the compilation of 100 questions that are part of InsightsIAS famous INSTA REVISION initiative for UPSC civil

INSTA 75 Days REVISION PLAN for Prelims 2020 - InstaTests

www.insightsonindia.com 78 Insights IAS

Solution: C

Republic

• A democratic polity can be classified into two categories–monarchy and republic. In

a monarchy, the head of the state (usually king or queen) enjoys a hereditary position,

that is, he comes into office through succession, e.g., Britain. In a republic, on the other

hand, the head of the state is always elected directly or indirectly for a fixed period,

e.g., USA.

• The term ‘republic’ in our Preamble indicates that India has an elected head called

the president. He is elected indirectly for a fixed period of five years.

• A republic also means two more things: one, vesting of political sovereignty in the

people and not in a single individual like a king; second, the absence of any privileged

class and hence all public offices being opened to every citizen without any

discrimination.

81. Which of the following categories are covered under the expression Union of India

under Article 1 of the Indian Constitution?

1. Territories of the states

2. Union territories

3. Territories that may be acquired by the Government of India at any time

Select the correct answer using the code given below:

(a) 1 only

(b) 2 and 3 only

(c) 1 and 2 only

(d) 1, 2 and 3

Solution: A

Article 1 describes India, that is, Bharat as a ‘Union of States’ rather than a ‘Federation of

States’. This provision deals with two things: one, name of the country, and two, type of polity

According to Article 1, the territory of India can be classified into three categories:

1. Territories of the states

2. Union territories

3. Territories that may be acquired by the Government of India at any time.

Page 81: SIMPLYFYING IAS EXAM PREPARATION - INSIGHTSIAS€¦ · This document is the compilation of 100 questions that are part of InsightsIAS famous INSTA REVISION initiative for UPSC civil

INSTA 75 Days REVISION PLAN for Prelims 2020 - InstaTests

www.insightsonindia.com 79 Insights IAS

Notably, the ‘Territory of India’ is a wider expression than the ‘Union of India’ because the

latter includes only states while the former includes not only the states, but also union

territories and territories that may be acquired by the Government of India at any future time.

The states are the members of the federal system and share a distribution of powers with the

Centre. The union territories and the acquired territories, on the other hand, are directly

administered by the Central government.

82. Consider the following statements regarding Economic Census

1. Economic census usually conducted along with the General Census in India

2. It is the census of all entrepreneurial units in the country either agricultural or

non-agricultural.

3. It is 100% centrally sponsored scheme of the Ministry of Statistics and Program

Implementation, Government of India.

Which of the statements given above is/are correct?

(a) 1 and 2 only

(b) 2 only

(c) 3 only

(d) 2 and 3 only

Solution: D

Economic Census gives the basic entrepreneurial data for planning and development in the

State specially for unorganized sectors of the economy.

• It is 100% centrally sponsored scheme of the Ministry of Statistics and Program

Implementation, Government of India.

• Five Economic Census have been conducted earlier throughout the country during the

year 1977, 1980, 1990, 1998 and 2005.

• Information collected under Economic Census provides updated frame for conducting

detailed follow up surveys.

Coverage of Economic Census

• Economic Census is the complete count of all entrepreneurial units located within the

geographical boundaries of the Country.

• Information on number of establishments and employment in all type of

establishments, unpaid/paid workers, female workers, child workers, ownership of

establishments, use of power, registration of establishments, source of finance etc. is

collected.

Page 82: SIMPLYFYING IAS EXAM PREPARATION - INSIGHTSIAS€¦ · This document is the compilation of 100 questions that are part of InsightsIAS famous INSTA REVISION initiative for UPSC civil

INSTA 75 Days REVISION PLAN for Prelims 2020 - InstaTests

www.insightsonindia.com 80 Insights IAS

• Complete address of enterprises having 10 or more workers is recorded.

http://esaharyana.gov.in/en-us/Economic-Census

83. Consider the following statements regarding qualifications to grant a certificate of

naturalization

1. He has either resided in India or been in the service of a Government in India or

partly the one and partly the other, throughout the period of twelve months

2. During the fourteen years immediately preceding the said period of twelve

months, he has either resided in India

3. He has an adequate knowledge of a language specified in the Eighth Schedule to

the Constitution

4. Government of India may waive all or any of the conditions for naturalization

Which of the statements given above is/are correct?

(a) 1, 2 and 3 only

(b) 2, 3 and 4 only

(c) 1, 3 and 4 only

(d) 1, 2, 3 and 4

Solution: D

By Naturalization

The Central Government may, on an application, grant a certificate of naturalization to any

person (not being an illegal migrant) if he possesses the following qualifications:

(a) that he is not a subject or citizen of any country where citizens of India are prevented

from becoming subjects or citizens of that country by naturalization;

(b) that, if he is a citizen of any country, he undertakes to renounce the citizenship of that

country in the event of his application for Indian citizenship being accepted;

(c) that he has either resided in India or been in the service of a Government in India or partly

the one and partly the other, throughout the period of twelve months immediately

preceding the date of the application;

(d) that during the fourteen years immediately preceding the said period of twelve months,

he has either resided in India or been in the service of a Government in India, or partly

the one and partly the other, for periods amounting in the aggregate to not less than

eleven years;

(e) that he is of good character;

(f) that he has an adequate knowledge of a language specified in the Eighth Schedule to

the Constitution; and

Page 83: SIMPLYFYING IAS EXAM PREPARATION - INSIGHTSIAS€¦ · This document is the compilation of 100 questions that are part of InsightsIAS famous INSTA REVISION initiative for UPSC civil

INSTA 75 Days REVISION PLAN for Prelims 2020 - InstaTests

www.insightsonindia.com 81 Insights IAS

(g) that in the event of a certificate of naturalization being granted to him, he intends to

reside in India, or to enter into or continue in, service under a Government in India or

under an international organization of which India is a member or under a society,

company or body of persons established in India. However, the Government of India may

waive all or any of the above conditions for naturalization in the case of a person who

has rendered distinguished service to the science, philosophy, art, literature, world peace

or human progress. Every naturalized citizen must take an oath of allegiance to the

Constitution of India.

84. Which of the following rights is/are considered as part of Article 21?

1. Right against tapping of telephonic conversation

2. Right to privacy

3. Right to travel abroad

4. Right to know about government activities

Select the correct answer using the code given below:

(a) 1 and 2 only

(b) 2 and 3 only

(c) 3 and 4 only

(d) 1 and 4 only

Solution: B

The Supreme Court has reaffirmed its judgement in the Menaka case in the subsequent

cases. It has declared the following rights as part of Article 21:

(1) Right to live with human dignity.

(2) Right to decent environment including pollution free water and air and protection against

hazardous industries.

(3) Right to livelihood.

(4) Right to privacy.

(5) Right to shelter.

(6) Right to health.

(7) Right to free education up to 14 years of age.

(8) Right to free legal aid.

(9) Right against solitary confinement.

(10) Right to speedy trial.

(11) Right against handcuffing.

(12) Right against inhuman treatment.

Page 84: SIMPLYFYING IAS EXAM PREPARATION - INSIGHTSIAS€¦ · This document is the compilation of 100 questions that are part of InsightsIAS famous INSTA REVISION initiative for UPSC civil

INSTA 75 Days REVISION PLAN for Prelims 2020 - InstaTests

www.insightsonindia.com 82 Insights IAS

(13) Right against delayed execution.

(14) Right to travel abroad.

(15) Right against bonded labour.

(16) Right against custodial harassment.

Freedom of Speech and Expression

It implies that every citizen has the right to express his views, opinions, belief and convictions

freely by word of mouth, writing, printing, picturing or in any other manner. The Supreme

Court held that the freedom of speech and expression includes the following:

(a) Right to propagate one’s views as well as views of others.

(b) Freedom of the press.

(c) Freedom of commercial advertisements.

(d) Right against tapping of telephonic conversation.

(e) Right to telecast, that is, government has no monopoly on electronic media.

(f) Right against bundh called by a political party or organization.

(g) Right to know about government activities.

(h) Freedom of silence.

(i) Right against imposition of pre-censorship on a newspaper.

(j) Right to demonstration or picketing but not right to strike.

85. Consider the following statements regarding prepaid payment instruments (PPIs)

1. They are instruments that facilitate purchase of goods and services, including

financial services, remittance facilities.

2. They facilitate the purchase of goods and services as well as inter-personal

remittance transactions such as sending money to a friend

Which of the statements given above is/are correct?

(a) 1 only

(b) 2 only

(c) Both 1 and 2

(d) Neither 1 nor 2

Solution: C

Page 85: SIMPLYFYING IAS EXAM PREPARATION - INSIGHTSIAS€¦ · This document is the compilation of 100 questions that are part of InsightsIAS famous INSTA REVISION initiative for UPSC civil

INSTA 75 Days REVISION PLAN for Prelims 2020 - InstaTests

www.insightsonindia.com 83 Insights IAS

Prepaid payment instruments (PPIs)

• PPIs are instruments that facilitate purchase of goods and services, including financial

services, remittance facilities, etc., against the value stored on such instruments.

• It comes with a pre-loaded value and in some cases a pre-defined purpose of

payment.

• They facilitate the purchase of goods and services as well as inter-personal remittance

transactions such as sending money to a friend or a family member.

• These payment instruments are licensed and regulated by the Reserve Bank of India

under the Payment and Settlement Systems Act, 2007.

86. Consider the following statements regarding Article 29 of the Indian Constitution:

1. Article 29 grants protection to both religious minorities as well as linguistic

minorities.

2. The Supreme Court held that the scope of this article is restricted to minorities

only.

Which of the statements given above is/are correct?

(a) 1 only

(b) 2 only

(c) Both 1 and 2

(d) Neither 1 nor 2

Solution: A

Protection of Interests of Minorities

• Article 29 provides that any section of the citizens residing in any part of India having

a distinct language, script or culture of its own, shall have the right to conserve the

same. Further, no citizen shall be denied admission into any educational institution

maintained by the State or receiving aid out of State funds on grounds only of religion,

race, caste, or language.

• The first provision protects the right of a group while the second provision guarantees

the right of a citizen as an individual irrespective of the community to which he

belongs.

• Article 29 grants protection to both religious minorities as well as linguistic

minorities. However, the Supreme Court held that the scope of this article is not

necessarily restricted to minorities only, as it is commonly assumed to be. This is

because of the use of words ‘section of citizens’ in the Article that include minorities

as well as majority.

Page 86: SIMPLYFYING IAS EXAM PREPARATION - INSIGHTSIAS€¦ · This document is the compilation of 100 questions that are part of InsightsIAS famous INSTA REVISION initiative for UPSC civil

INSTA 75 Days REVISION PLAN for Prelims 2020 - InstaTests

www.insightsonindia.com 84 Insights IAS

The Supreme Court also held that the right to conserve the language includes the right to

agitate for the protection of the language. Hence, the political speeches or promises made

for the conservation of the language of a section of the citizens does not amount to corrupt

practice under the Representation of the People Act, 1951.

87. Consider the following statements regarding Quo-Warranto writ

1. It can be issued in cases of ministerial office.

2. This writ can be sought by any interested person and not necessarily by the

aggrieved person.

Which of the statements given above is/are correct?

(a) 1 only

(b) 2 only

(c) Both 1 and 2

(d) Neither 1 nor 2

Solution: B

Quo-Warranto

• In the literal sense, it means ‘by what authority or warrant’. It is issued by the court

to enquire into the legality of claim of a person to a public office. Hence, it prevents

illegal usurpation of public office by a person.

• The writ can be issued only in case of a substantive public office of a permanent

character created by a statute or by the Constitution. It cannot be issued in cases of

ministerial office or private office. Unlike the other four writs, this can be sought by

any interested person and not necessarily by the aggrieved person.

88. Voluntary Retention Route, sometime seen in the news, is related to

(a) Giving up of subsidies

(b) Foreign portfolio investment

(c) Allocation of government contracts

(d) None of the above

Solution: B

Page 87: SIMPLYFYING IAS EXAM PREPARATION - INSIGHTSIAS€¦ · This document is the compilation of 100 questions that are part of InsightsIAS famous INSTA REVISION initiative for UPSC civil

INSTA 75 Days REVISION PLAN for Prelims 2020 - InstaTests

www.insightsonindia.com 85 Insights IAS

Voluntary Retention Route is a scheme to encourage Foreign Portfolio Investors (FPIs) to

undertake long-term investments in Indian debt markets.

• The minimum retention period would be three years. During this period, FPIs shall

maintain a minimum of 75 per cent of the allocated amount in India.

• FPIs registered with Securities and Exchange Board of India (SEBI) are eligible to

voluntarily invest through the route in government and corporate bonds.

• Allocation of investment amount to FPIs under this route shall be made on tap or

through auctions.

89. Which of the following Directives is/are directives provided outside the Directives

Principles of State Policy (DPSP) included in Part IV of the Indian Constitution?

1. Claims of SCs and STs to Services

2. Instruction in mother tongue

3. Development of the Hindi Language

Select the correct answer using the code given below:

(a) 1 and 2 only

(b) 2 and 3 only

(c) 1 and 3 only

(d) 1, 2 and 3

Solution: D

Apart from the Directives included in Part IV, there are some other Directives contained in

other Parts of the Constitution. They are:

• Claims of SCs and STs to Services: The claims of the members of the Scheduled Castes

and the Scheduled Tribes shall be taken into consideration, consistently with the

maintenance of efficiency of administration, in the making of appointments to

services and posts in connection with the affairs of the Union or a State (Article 335 in

Part XVI).

• Instruction in mother tongue: It shall be the endeavor of every state and every local

authority within the state to provide adequate facilities for instruction in the mother

tongue at the primary stage of education to children belonging to linguistic minority

groups (Article 350-A in Part XVII).

• Development of the Hindi Language: It shall be the duty of the Union to promote the

spread of the Hindi language and to develop it so that it may serve as a medium of

expression for all the elements of the composite culture of India (Article 351 in Part

XVII).

Page 88: SIMPLYFYING IAS EXAM PREPARATION - INSIGHTSIAS€¦ · This document is the compilation of 100 questions that are part of InsightsIAS famous INSTA REVISION initiative for UPSC civil

INSTA 75 Days REVISION PLAN for Prelims 2020 - InstaTests

www.insightsonindia.com 86 Insights IAS

• The above Directives are also non-justiciable in nature. However, they are also given

equal importance and attention by the judiciary on the ground that all parts of the

constitution must be read together.

90. Consider the following statements regarding the characteristics of the Fundamental

Duties

1. All of the fundamental duties are moral duties.

2. Fundamental Duties are confined to citizens only and do not extend to foreigners.

3. Parliament is free to enforce Fundamental Duties by suitable legislation.

Which of the statements given above is/are correct?

(a) 1 and 2 only

(b) 2 and 3 only

(c) 3 only

(d) 2 only

Solution: B

Following points can be noted with regard to the characteristics of the Fundamental Duties:

• Some of them are moral duties while others are civic duties. For instance, cherishing

noble ideals of freedom struggle is a moral precept and respecting the Constitution,

National Flag and National Anthem is a civic duty.

• They refer to such values which have been a part of the Indian tradition, mythology,

religions and practices. In other words, they essentially contain just a codification of

tasks integral to the Indian way of life.

• Unlike some of the Fundamental Rights which extend to all persons whether citizens

or foreigners, the Fundamental Duties are confined to citizens only and do not extend

to foreigners.

• Like the Directive Principles, the fundamental duties are also non-justiciable. The

Constitution does not provide for their direct enforcement by the courts. Moreover,

there is not legal sanction against their violation. However, the Parliament is free to

enforce them by suitable legislation.

91. Consider the following statements regarding National Common Mobility Card

1. It is the Indigenous Automatic Fare Collection System

2. National Common Mobility Card (NCMC) – One Nation, One Card for transport

mobility is an initiative of Ministry of Road Transport and Highways.

Page 89: SIMPLYFYING IAS EXAM PREPARATION - INSIGHTSIAS€¦ · This document is the compilation of 100 questions that are part of InsightsIAS famous INSTA REVISION initiative for UPSC civil

INSTA 75 Days REVISION PLAN for Prelims 2020 - InstaTests

www.insightsonindia.com 87 Insights IAS

3. The cards can be issued by public sector banks only.

Which of the statements given above is/are correct?

(a) 2 and 3 only

(b) 1 only

(c) 1 and 2 only

(d) 1, 2 and 3

Solution: B

• Prime Minister Narendra Modi recently launched a National Common Mobility Card

(NCMC).

• National Common Mobility Card (NCMC) – One Nation, One Card for transport

mobility is an initiative of the Ministry of Housing and Urban Affairs to enable

seamless travel by different metros and other transport systems across the country

besides retail shopping and purchases.

• The Indigenous Automatic Fare Collection System based on One Nation One Card

Model is the first of its kind in India.

Key features:

• Dubbed as ‘One Nation One Card’, the inter-operable transport card would allow the

holders to pay for their bus travel, toll taxes, parking charges, retail shopping and even

withdraw money.

• This card runs on RuPay card. With the advent of this indigenously-developed and one-

of-its-kind card, the country is no longer required to be dependent on foreign

technology.

• The stored value on card supports offline transaction across all travel needs with

minimal financial risk to involved stakeholders.

• It is an open loop card, which can be used for all local travel needs in the country.

• The cards can be issued by all public and private banks the same way credit, debit, and

prepaid cards are issued.

https://vikaspedia.in/e-governance/online-citizen-services/government-to-citizen-services-

g2c/transport-related-services/national-common-mobility-card

92. Consider the following statements regarding procedure for the amendment to the

Constitution of India

1. The bill does not require prior permission of the president.

2. The bill can be introduced only by the minister.

Page 90: SIMPLYFYING IAS EXAM PREPARATION - INSIGHTSIAS€¦ · This document is the compilation of 100 questions that are part of InsightsIAS famous INSTA REVISION initiative for UPSC civil

INSTA 75 Days REVISION PLAN for Prelims 2020 - InstaTests

www.insightsonindia.com 88 Insights IAS

3. The president can give his assent to the bill or exercise veto powers.

Which of the statements given above is/are correct?

(a) 1 only

(b) 2 and 3 only

(c) 1 and 2 only

(d) 1, 2 and 3

Solution: A

The procedure for the amendment of the Constitution as laid down in Article 368 is as

follows:

• An amendment of the Constitution can be initiated only by the introduction of a bill

for the purpose in either House of Parliament and not in the state legislatures.

• The bill can be introduced either by a minister or by a private member and does not

require prior permission of the president.

• The bill must be passed in each House by a special majority, that is, a majority of the

total membership of the House and a majority of two-thirds of the members of the

House present and voting.

• Each House must pass the bill separately. In case of a disagreement between the two

Houses, there is no provision for holding a joint sitting of the two Houses for the

purpose of deliberation and passage of the bill.

• If the bill seeks to amend the federal provisions of the Constitution, it must also be

ratified by the legislatures of half of the states by a simple majority, that is, a majority

of the members of the House present and voting.

• After duly passed by both the Houses of Parliament and ratified by the state

legislatures, where necessary, the bill is presented to the president for assent.

• The president must give his assent to the bill. He can neither withhold his assent to

the bill nor return the bill for reconsideration of the Parliament.

• After the president’s assent, the bill becomes an Act (i.e., a constitutional amendment

act) and the Constitution stands amended in accordance with the terms of the Act.

93. Consider the following statements regarding parliamentary system of government in

India and Britain

1. ‘Shadow cabinet’ is a unique institution of the British cabinet system.

2. Britain has the system of legal responsibility of the minister while India has no

such system.

Page 91: SIMPLYFYING IAS EXAM PREPARATION - INSIGHTSIAS€¦ · This document is the compilation of 100 questions that are part of InsightsIAS famous INSTA REVISION initiative for UPSC civil

INSTA 75 Days REVISION PLAN for Prelims 2020 - InstaTests

www.insightsonindia.com 89 Insights IAS

3. Both in Britain and in India, the prime minister may be a member of any of the

two Houses of Parliament.

Which of the statements given above is/are correct?

(a) 1 only

(b) 1 and 2 only

(c) 3 only

(d) 2 and 3 only

Solution: B

The parliamentary system of government in India is largely based on the British parliamentary

system. However, it never became a replica of the British system and differs in the following

respects:

• India has a republican system in place of British monarchical system. In other words,

the Head of the State in India (that is, President) is elected, while the Head of the State

in Britain (that is, King or Queen) enjoys a hereditary position.

• The British system is based on the doctrine of the sovereignty of Parliament, while the

Parliament is not supreme in India and enjoys limited and restricted powers due to a

written Constitution, federal system, judicial review and fundamental rights.

• In Britain, the prime minister should be a member of the Lower House (House of

Commons) of the Parliament. In India, the prime minister may be a member of any of

the two Houses of Parliament.

• Usually, the members of Parliament alone are appointed as ministers in Britain. In

India, a person who is not a member of Parliament can also be appointed as minister,

but for a maximum period of six months.

• Britain has the system of legal responsibility of the minister while India has no such

system. Unlike in Britain, the ministers in India are not required to countersign the

official acts of the Head of the State.

• ‘Shadow cabinet’ is an unique institution of the British cabinet system. It is formed by

the opposition party to balance the ruling cabinet and to prepare its members for

future ministerial office. There is no such institution in India.

94. Consider the following statements regarding Index of Industrial Production (IIP)

1. It is published by Central Statistics Office(CSO)

2. It is published annually with two months lag.

3. It is a long term indicator of industrial growth.

Which of the statements given above is/are correct?

Page 92: SIMPLYFYING IAS EXAM PREPARATION - INSIGHTSIAS€¦ · This document is the compilation of 100 questions that are part of InsightsIAS famous INSTA REVISION initiative for UPSC civil

INSTA 75 Days REVISION PLAN for Prelims 2020 - InstaTests

www.insightsonindia.com 90 Insights IAS

(a) 2 and 3 only

(b) 1 and 3 only

(c) 1 only

(d) 1, 2 and 3

Solution: C

• Index of Industrial Production (IIP) is an index which helps us understand the growth

of various sectors in the Indian economy such as mining, electricity and

manufacturing. It is a short term indicator of industrial growth.

• Index of Industrial Production (IIP) is released by the Central Statistics Office (CSO) of

the Ministry of Statistics and Programme Implementation. IIP is published monthly,

with six weeks delay.

It covers 865 (Older series 682) items comprising:

• Manufacturing (809 items, Older series 620 items),

• Mining (55 items, Older Series 61 items) &

• Electricity (1 item).

The weights of the three sectors are:

• Manufacturing – 77.63%,

• Mining – 14.37%,

• Electricity – 7.99%

95. Consider the following statements regarding Allocation of Taxing Powers in Indian

federation

1. The residuary power of taxation is vested in the Parliament.

2. No taxes have been imposed under the residuary power of parliament.

Which of the statements given above is/are correct?

(a) 1 only

(b) 2 only

(c) Both 1 and 2

(d) Neither 1 nor 2

Solution: A

Page 93: SIMPLYFYING IAS EXAM PREPARATION - INSIGHTSIAS€¦ · This document is the compilation of 100 questions that are part of InsightsIAS famous INSTA REVISION initiative for UPSC civil

INSTA 75 Days REVISION PLAN for Prelims 2020 - InstaTests

www.insightsonindia.com 91 Insights IAS

Allocation of Taxing Powers

The Constitution divides the taxing powers between the Centre and the states in the following

way:

• The Parliament has exclusive power to levy taxes on subjects enumerated in the Union

List (which are 13 in number).

• The state legislature has exclusive power to levy taxes on subjects enumerated in the

State List (which are 18 in number).

• There are no tax entries in the Concurrent List. In other words, the concurrent

jurisdiction is not available with respect to tax legislation. But, the 101st Amendment

Act of 2016 has made an exception by making a special provision with respect to goods

and services tax. This Amendment has conferred concurrent power upon Parliament

and State Legislatures to make laws governing goods and services tax.

• The residuary power of taxation (that is, the power to impose taxes not enumerated

in any of the three lists) is vested in the Parliament. Under this provision, the

Parliament has imposed gift tax, wealth tax and expenditure tax.

96. Nalsarovar Bird Sanctuary is located in

(a) Arunachal Pradesh

(b) Uttar Pradesh

(c) Gujarat

(d) Maharashtra

Solution: C

• Nal Sarovar is a Bird Sanctuary, spread in the area of 120.82-sq-km Lake, a serene

marshland with shallow waters (4-5 ft) which contains 36 small islands. Nalsarovar-

Ahmedabad distance is about 60 km located near Sanand Village, in Gujarat. Mainly

inhabited by migratory birds as their wintering ground, Nalsarovar wetland is the

largest wetland bird sanctuary in Gujarat, and one of the largest in India. Nalsarovar is

a Bird Sanctuary since April 1969 and Nalsarovar is a Ramsar site since 24 September

2012.

• The Nal Sarovar Lake is in a lowest-lying area between the Central Gujarat and East

Saurashtra. It represents a sea link that once existed between the Little Rann and the

Gulf of Khamba.

Page 94: SIMPLYFYING IAS EXAM PREPARATION - INSIGHTSIAS€¦ · This document is the compilation of 100 questions that are part of InsightsIAS famous INSTA REVISION initiative for UPSC civil

INSTA 75 Days REVISION PLAN for Prelims 2020 - InstaTests

www.insightsonindia.com 92 Insights IAS

97. Consider the following statements regarding roles of Directives Principles of State Policy

(DPSP)

1. They facilitate stability and continuity in domestic and foreign policies.

2. They are supplementary to the fundamental rights of the citizens.

3. They serve as a crucial test for the performance of the government.

Which of the statements given above is/are correct?

(a) 1 and 2 only

(b) 2 and 3 only

(c) 1 and 3 only

(d) 1, 2 and 3

Solution: D

The Directives also play the following roles:

• They facilitate stability and continuity in domestic and foreign policies in political,

economic and social spheres in spite of the changes of the party in power.

• They are supplementary to the fundamental rights of the citizens. They are intended

to fill in the vacuum in Part III by providing for social and economic rights.

• Their implementation creates a favorable atmosphere for the full and proper

enjoyment of the fundamental rights by the citizens. Political democracy, without

economic democracy, has no meaning.

• They enable the opposition to exercise influence and control over the operations of

the government. The Opposition can blame the ruling party on the ground that its

activities are opposed to the Directives.

• They serve as a crucial test for the performance of the government. The people can

examine the policies and programmes of the government in the light of these

constitutional declarations.

• They serve as common political manifesto. ‘A ruling party, irrespective of its political

ideology, has to recognise the fact that these principles are intended to be its guide,

philosopher and friend in its legislative and executive acts

98. Consider the following statements regarding International Atomic Energy Agency (IAEA)

1. It aims to promote the peaceful use of nuclear energy, and to inhibit its use for

any military purpose, including nuclear weapons.

2. It reports annually to the UN General Assembly

Which of the statements given above is/are correct?

Page 95: SIMPLYFYING IAS EXAM PREPARATION - INSIGHTSIAS€¦ · This document is the compilation of 100 questions that are part of InsightsIAS famous INSTA REVISION initiative for UPSC civil

INSTA 75 Days REVISION PLAN for Prelims 2020 - InstaTests

www.insightsonindia.com 93 Insights IAS

(a) 1 only

(b) 2 only

(c) Both 1 and 2

(d) Neither 1 nor 2

Solution: C

International Atomic Energy Agency (IAEA)

• It is set up as the world’s “Atoms for Peace” organization in 1957 within the United

Nations family.

• Headquarters in Vienna, Austria.

• It is an independent international organization that reports annually to the UN General

Assembly. When necessary, the IAEA also reports to the UN Security Council in regards

to instances of members’ non-compliance with safeguards and security obligations.

Functions:

• Works with its Member States and multiple partners worldwide to promote the safe,

secure and peaceful use of nuclear technologies.

• Seeks to promote the peaceful use of nuclear energy, and to inhibit its use for any

military purpose, including nuclear weapons.

99. Consider the following statements regarding freedom of trade, commerce and

intercourse provided under Article 301 of the Indian Constitution:

1. The freedom is confined to interstate trade, commerce and intercourse but does

not extend to intra-state trade, commerce and intercourse.

2. Only Parliament can impose restrictions on the freedom of trade, commerce and

intercourse between the states or within a state.

Which of the statements given above is/are correct?

(a) 1 only

(b) 2 only

(c) Both 1 and 2

(d) Neither 1 nor 2

Solution: D

Page 96: SIMPLYFYING IAS EXAM PREPARATION - INSIGHTSIAS€¦ · This document is the compilation of 100 questions that are part of InsightsIAS famous INSTA REVISION initiative for UPSC civil

INSTA 75 Days REVISION PLAN for Prelims 2020 - InstaTests

www.insightsonindia.com 94 Insights IAS

• Article 301 declares that trade, commerce and intercourse throughout the territory

of India shall be free. The object of this provision is to break down the border barriers

between the states and to create one unit with a view to encourage the free flow of

trade, commerce and intercourse in the country. The freedom under this provision is

not confined to interstate trade, commerce and intercourse but also extends to intra-

state trade, commerce and intercourse. Thus, Article 301 will be violated whether

restrictions are imposed at the frontier of any state or at any prior or subsequent

stage.

• The freedom guaranteed by Article 301 is a freedom from all restrictions, except those

which are provided for in the other provisions (Articles 302 to 305) of Part XIII of the

Constitution itself.

These are explained below:

(i) Parliament can impose restrictions on the freedom of trade, commerce

and intercourse between the states or within a state, in public interest. But,

the Parliament cannot give preference one state over another or

discriminate between the state except in the case of scarcity of goods in

any part of India.

(ii) The legislature of a state can imposes reasonable restrictions the freedom

of trade, commerce and intercourse with that state or within that state in

public interest. But, a bill for this purpose can be introduced in the

legislature only with the previous sanction of the president. Further, the

state legislature can give preference to one state over another or

discriminate between the states.

(iii) The legislature of a state can impose on goods imported from other states

or the union territories any tax to which similar goods manufactured in that

state are subject. This provision prohibits the imposition of discriminatory

taxes by the state.

(iv) The freedom (under Article 301) is subject to the nationalization laws (i.e.,

laws providing for monopolies in favour of the Center or the states). Thus,

the Parliament or the state legislature can make laws for the carrying on

by the respective government.

100. Consider the following statements regarding Small satellite launch vehicle (SSLV) built

by ISRO:

1. It will be used exclusively for nanosats and cubesats.

2. These are assembled only in an upright position in the assembly same like PSLV

and GSLV

3. SSLV rockets are capable of putting a payload of 500 kg to mid-inclination LEO.

Which of the statements given above is/are correct?

Page 97: SIMPLYFYING IAS EXAM PREPARATION - INSIGHTSIAS€¦ · This document is the compilation of 100 questions that are part of InsightsIAS famous INSTA REVISION initiative for UPSC civil

INSTA 75 Days REVISION PLAN for Prelims 2020 - InstaTests

www.insightsonindia.com 95 Insights IAS

(a) 1 and 3 only

(b) 2 and 3 only

(c) 3 only

(d) 1 and 3 only

Solution: A

SSLV is a new mini-rocket launcher designed and built by the Indian Space Research

Organization (ISRO) for launching small satellites. SSLV rockets are capable of putting a

payload of 500 kg to mid-inclination Low Earth Orbit (LEO) and 300 kg to Sun Synchronous

Orbit (SSO), making it ideal for launching small satellites.

SSLV is perfectly suited for launching multiple micro-satellites at a time and supports

multiple orbital drop-offs. Unlike the PSLV and GSLV, the SSLV can be assembled both

vertically and horizontally. It will be used exclusively for small satellites such as nanosats

and cubesats.